🔙 Quay lại trang tải sách pdf ebook Khám Phá Tư Duy Kỹ Thuật Giải Bất Đẳng Thức Bài Toán Min – Max Ebooks Nhóm Zalo ÑAËNG THAØNH NAM (Trung taâm Nghieân cöùu vaø phaùt trieån saûn phaåm giaùo duïc Newstudy.vn) SOAÏN THEO CAÁU TRUÙC MÔÙI AÙP DUÏNG KÌ THI THPT QUOÁC GIA (PHIEÂN BAÛN MÔÙI NHAÁT) Daønh cho hoïc sinh 10, 11, 12 naâng cao kieán thöùc. Boài döôõng hoïc sinh gioûi luyeän thi Quoác Gia. NHAØ XUAÁT BAÛN ÑAÏI HOÏC QUOÁC GIA HAØ NOÄI MUÏC LUÏC Chöông 1: Baát ñaúng thöùc vaø caùc kyõ thuaät cô baûn Chuû ñeà 1. Kyõ thuaät bieán ñoåi töông ñöông .............................................. 04 Chuû ñeà 2. Kyõ thuaät minh phaûn chöùng .................................................... 45 Chuû ñeà 3. Kyõ thuaät quy naïp toaùn hoïc..................................................... 56 Chuû ñeà 4. Kyõ thuaät mieàn giaù trò ............................................................. 60 Chuû ñeà 5. Kyõ thuaät söû duïng nguyeân lí Diricle ....................................... 68 Chuû ñeà 6. Kyõ thuaät tam thöùc baäc hai ..................................................... 73 Chuû ñeà 7. Kyõ thuaät ñaùnh giaù baát ñaúng thöùc tích phaân ......................... 93 Chöông 2: Baát ñaúng thöùc vaø phöông phaùp tieáp caän Chuû ñeà 1. Caùc kyõ thuaät söû suïng baát ñaúng thöùc AM-GM cô baûn.......... 102 Chuû ñeà 2. Kyõ thuaät gheùp caëp trong chöùng minh ñaúng thöùc AM-GM............. 198 Chuû ñeà 3. Kyõ thuaät söû duïng baát ñaúng thöùc AM-GM daïng coäng maãu soá ........211 Chuû ñeà 4. Kyõ thuaät söû duïng baát ñaúng thöùc Cauchy-Schwarz .............. 218 Chuû ñeà 5. Kyõ thuaät söû duïng baát ñaúng thöùc Cauchy-Schwarz daïng phaân thöùc ................................................................................ 243 Chuû ñeà 6. Kyõ thuaät tham soá hoùa........................................................... 278 Chuû ñeà 7. Baát ñaúng thöùc Holder vaø öùng duïng...................................... 291 Chuû ñeà 8. Kyõ thuaät söû duïng baát ñaúng thöùc Chebyshev ....................... 304 Chuû ñeà 9. Baát ñaúng thöùc Bernoulli vaø öùng duïng.................................. 314 Chöông 3: Phöông trình haøm soá trong giaûi toaùn baát ñaúng thöùc vaø cöïc trò Chuû ñeà 1. Kyõ thuaät söû duïng tính ñôn ñieäu vôùi baøi toaùn cöïc trò vaø baát ñaúng thöùc moät bieán soá ......................................................... 325 Chuû ñeà 2. Kyõ thuaät söû duïng tính ñôn ñieäu cho baøi toaùn cöïc trò vaø baát ñaúng thöùc hai bieán soá .......................................................... 351 Chuû ñeà 3. Kyõ thuaät söû duïng tính ñôn ñieäu cho baøi toaùn cöïc trò vaø baát ñaúng thöùc ba bieán soá............................................................ 379 Chuû ñeà 4. Kyõ thuaät söû duïng tính thuaàn nhaát....................................... 427 Chuû ñeà 5. Kyõ thuaät söû duïng baát ñaúng thöùc tieáp tuyeán ........................ 484 Chuû ñeà 6. Kyõ thuaät khaûo saùt haøm nhieàu bieán...................................... 502 Chuû ñeà 7. Kyõ thuaät söû duïng tính chaát cuûa nhò thöùc baäc nhaát vaø tam thöùc baäc hai.......................................................................... 534 Chuû ñeà 8. Baát ñaúng thöùc phuï ñaâng chuù yù vaø aùp duïng giaûi ñeà thi tuyeån sinh .. 540 Chuû ñeà 9. Baøi toaùn choïn loïc baát ñaúng thöùc vaø cöïc trò ba bieán............ 617 Chöông 4: Soá phöông phaùp chöùng minh baát ñaúng thöùc khaùc Chuû ñeà 1. Kyõ thuaät löôïng giaùc hoùa ....................................................... 654 Chuû ñeà 2. Kyõ thuaät söû duïng baát ñaúng thöùc Schur................................ 684 Chuû ñeà 3. Kyõ thuaät doàn bieán................................................................. 694 Cty TNHH MTV DVVH Khang Việt Chương 1: BẤT ĐẲNG THỨC VÀ CÁC KỸ THUẬT CƠ BẢN KIẾN THỨC CƠ BẢN VỀ BẤT ĐẲNG THỨC I. Định nghĩa bất đẳng thức Giả sử A và B là hai biểu thức bằng chữ hoặc bằng số. + A B ≥ (hoặc B A ≤ ) , A B ≤ (hoặc B A ≥ ) được gọi là các bất đẳng thức. + AB AB AB AB ≥ ⇔ − ≥ − ≥⇔ ≥ 0; 0 . + Một bất đẳng thức có thể đúng hoặc sai và ta quy ước khi nói về một bất đẳng thức mà không nói gì thêm thì ta hiểu đó là bất đẳng thức đúng. II. Tính chất cơ bản của bất đẳng thức ▪ ∀∈ ≥ a aa ; . ▪  ≤  ⇒ ≤ a ba c b c .  ≤ ▪ ∀ ∈ ≤⇒± ≤± abm a b a m b m ,, ;  . ▪  ≤  ⇒+≤+ a bacbd c d .  ≤ ▪ abc acb ≥+⇔ −≥ . ▪ khi m >0 ma mb  ≤ ab a b . ∀ ∈ ≤⇔  ≥ ,, ; ma mb khi m < 0 a b  ≤  ∀ ∈ ≤⇔  ≥  khi m >0 m m ab a ba b + . ▪ ,, ;  khi m < 0 m m ▪ Nếu 1 1 a b >>⇒ < 0a b . ▪ ,,, ; +  ≥ a c b d . ∀ ∈ ⇒≥  ≥ a b c d ab cd n n  ≥ a b . ▪ 0 , ab n ≥ ≥ ⇒ ∀∈  ≥ n n a b x y  >⇒ > a aa ▪ 1 . x y 0; > > < <⇒ <  x y a aa 0 1 ▪ 21 21 21 21 ; , + + + + > ⇒ > > ∀∈ n n n n ab a b a b n . 3 Khám phá tư duy Kỹ thuật giải bất ĐT Bài toán Max – Min – Đặng Thành Nam 1. Bất đẳng thức về giá trị tuyệt đối ▪ − ≤ ≤ ∀∈ aaa a ,  . ▪ a a khi < ⇔− < < > α α α ( ) 0 α . α ▪ ( ) 0  > a α α α . a khi >⇔ >  < − a ▪ a b a b a b ab − ≤+≤ + ∀ ∈ , , ( )  . 2. Bất đẳng thức liên quan đến hàm số mũ và logarit ▪ 1 01   > <<   ⇒> ⇒< xy xy a a xy xy . ; aa aa   > > a a ▪ 1 01   > <<   ⇒> ⇒< xy xy . xy xy log log ; log log 0 0   >> >> aa aa 3. Bất đẳng thức AM – GM Cho n số thực không âm 1 2 , ,..., n aa a ta có ... ... + ++≥ n n n aa aaa a 1 21 2 n . Đẳng thức xảy ra khi và chỉ khi 1 2 = = = ... n aa a . 4. Bất đẳng thức Cauchy – Schwarz Cho 2 dãy số thực ( ) ( ) 12 12 , ,..., ; , ,..., n n aa a bb b ta có ( ) ( )( ) 2 2 2 222 2 11 2 2 1 2 1 2 + ++ ≤ + ++ + ++ ... ... ... nn n n ab a b a b a a a b b b . Đẳng thức xảy ra khi và chỉ khi a kb i n k i i = = ∈ , 1, , .  CHỦ ĐỀ 1: KỸ THUẬT BIẾN ĐỔI TƯƠNG ĐƯƠNG Xuất phát từ bất đẳng thức cơ bản và hết sức tự nhiên 2 x AB ≥ −≥ 0; 0 với mọi số thực x ta có các bất đẳng thức hết sức đẹp mắt. Nội dung chủ đề này đề cập đến kỹ năng biến đổi bất đẳng thức về dạng luôn đúng. Các bài toán đề cập đến là các bài toán trong chủ đề này các bạn chú ý sẽ được sử dụng đến trong các chủ đề khác ở các chương sau như một bài toán phụ. A. NỘI DUNG PHƯƠNG PHÁP I. Các bất đẳng thức cơ bản Bình phương của một số thực Với mọi số thực x ta luôn có 2 x ≥ 0. Đẳng thức xảy ra khi và chỉ khi x = 0 . Từ đó ta có các bất đẳng thức với 2 biến và 3 biến thường sử dụng như sau: ▪ ( )2 a b − ≥ 0 hay 2 2 a b ab + ≥ 2 . Đẳng thức xảy ra khi và chỉ khi a b = . 4 Cty TNHH MTV DVVH Khang Việt ▪ ( ) ( ) ( ) 222 ab bc ca − +− +− ≥ 0 hay 222 a b c ab bc ca ++≥++ hoặc ( ) ( ) 2 a b c ab bc ca ++ ≥ + + 3 hoặc ( ) ( ) 222 2 3 a b c abc + + ≥ ++ . Đẳng thức xảy ra khi và chỉ khi abc = = . Bất đẳng thức về trị tuyệt đối ▪ Với 2 số thực x,y ta luôn có x y xy + ≥+ . Đẳng thức xảy ra khi và chỉ khi xy ≥ 0 . ▪ Với 2 số thực x,y ta luôn có xy x y −≥ − . Đẳng thức xảy ra khi và chỉ khi xx y ( ) − ≥ 0 . Bất đẳng thức về độ dài cạnh của một tam giác ▪ a b cb c ac a b +> +> +> ; ; . ▪ a b cb c ac a b >− >− >− ; ; . ▪ ( ) 222 a b c ab bc ca ++< ++ 2 . II. Một số hằng đẳng thức cần lưu ý ▪ ( )( ) 333 222 a b c abc a b c a b c ab bc ca + + − = ++ + + − − − 3 . ▪ ( ) ( )( )( ) 3 333 abc a b c abbcca ++ = + + + + + + 3 . ▪ ( )( )( ) ( )( ) a b b c c a a b c ab bc ca abc + + + = ++ + + − . ▪ −−− ( )( )( ) −−− + + =− ab bc ca abbcca c a b abc . ▪ .. 0 − − − −−− +++ = a b b c c a a bb cc a a b b c a c a bb cc a . + + + +++ 1) Kỹ thuật biến dùng định nghĩa Để chứng minh bất đẳng thức: A B ≥ . Ta chứng minh bất đẳng thức A B − ≥ 0 đúng. 2 2 2 Ví dụ 1. Cho x > y và xy = 1. Chứng minh rằng ( ) x y +≥ . ( ) x y − Lời giải Ta có ( ) ( ) 2 2 2 2 2 8 x y x y xy x y + =− + =− + 2 2 (vì xy = 1) ⇒ ( ) ( ) ( ) 2 2 2 4 2 x y xy xy + =− + − + 4. 4 . Do đó BĐT cần chứng minh tương đương với ( ) ( ) ( ) 42 2 xy xy xy − + − +≥ − 4 4 8. 5 Khám phá tư duy Kỹ thuật giải bất ĐT Bài toán Max – Min – Đặng Thành Nam ⇔ ( ) ( ) 4 2 xy xy − − − +≥ 4 40 ⇔ ( )2 2 2 0   −−≥   x y . Bất đẳng thức cuối đúng nên ta có điều phải chứng minh. Ví dụ 2. a) Cho x,y là hai số thực thoả mãn điều kiện xy ≥1 . Chứng minh rằng 112 + + x y + xy . + ≥ 2 2 1 1 1 b) Cho a,b,c là các số thực không nhỏ hơn 1 chứng minh 111 3 +++ abc + abc . ++≥ 333 111 1 c) Cho xyz , , 0;1 ∈[ ]. Tìm giá trị lớn nhất của biểu thức   111 1111 ( ) 3 33 P xyzxyz . =+ + +     +++ Lời giải a) Bất đẳng thức đã cho tương đương với:     11 11 0   −+ −≥     + + + +   x y xy xy 2 2 1 1 1 1 2 2 xy x xy y − − + ≥ ⇔( ) ( ) ( ) ( ) 2 20 1 .1 1 .1 ++ ++ x xy y xy xy x yx y () () 0 − − + ≥ ⇔ ( ) ( ) ( ) ( ) 2 2 1 .1 1 .1 ++ ++ x xy y xy 2 ⇔ ( ) ( ) y x xy − − ≥ 2 2 10 ( ) ( ) ( ) +++ 1 .1 .1 x y xy BĐT cuối này đúng do xy ≥1. Vậy ta có đpcm. Đẳng thức xảy ra khi và chỉ khi x y = hoặc xy =1. b) Sử dụng bất đẳng thức: 112 , 1 ( ) 2 2 x y xy . + + +xy +≥ ≥ 1 1 1 11 2 Ta có 3 3 3 3 + ≥ 1 1 1 + + a b + a b 6 Cty TNHH MTV DVVH Khang Việt 11 2 + ≥ 3 4 1 1 1 + + + c abc abc .     +≥ =+   + + + 11 2 4 2 2.1 1 1 1 . abc a b abc a b abc 33 4 33 4 Cộng theo vế ba bất đẳng thức trên ta có đpcm. Chú ý. Bất đẳng thức này được áp dụng khá phổ biến trong một số bài toán cực trị. Một số dạng tương tự bất đẳng thức trên như sau 112 ,1 1 ▪ ( ) 2 2 x y xy . + + +xy + ≤ −< ≤ 1 1 1 112 , 1 ▪ ( ) 2 2 xy x y. +≥ ≥ + + +xy 1 1 1 112 ,1 1 ▪ ( ) 2 2 xy x y. + + +xy + ≤ −< ≤ 1 1 1 11 2 c) Sử dụng kết quả bài toán trên ta có : 3 3 3 3 + ≤ 1 1 1 + + x y + x y 11 2 + ≤ 3 4 1 1 1 + + + z xyz xyz 22 4 4 +≤ =+ + + + xyz x y xyz xyz 1 1 1 1 33 4 4 44 Cộng theo vế các bất đẳng thức trên ta suy ra   111 3 111 1 3 ( ) 3 33 3 33 + + ≤ ⇒=+ + + ≤   P xyz 111 111 1 +++ +++ +   xyz xyz xyz Đẳng thức xảy ra khi và chỉ khi xyz = = Vậy giá trị lớn nhất của P = 3 . Ví dụ 3. Chứng minh rằng với mọi số thực x,y ta có ( )( ) ( )2   + +   + +≥x y 2 23 1 x y . 1 14 Lời giải 2 2 4 2 2 2 2 2 1 Chú ý: ( )( ) ( ) ( ) ( ) ( ) + + =+ + + ≥+ + xy x y − +− x y xy xy . 11 1 1 3 3 7 Khám phá tư duy Kỹ thuật giải bất ĐT Bài toán Max – Min – Đặng Thành Nam Bất đẳng thức được chứng minh. Đẳng thức xảy ra khi và chỉ khi 12 x y = = ± . Ví dụ 4. Chứng minh rằng với mọi số thực a,b không âm thoả mãn a b, 1; < 2 12 12 1 4 a b + ≥ ta có ( )( ) − −   − − ≥   − − −−   3 ,2 a b a b a b ab . ( )( ) 11 2 Lời giải Bất đẳng thức đã cho tương đương với: 2 ( ) ( ) − +−≥ − − −− ab a b 2230 . 2 ( )( )( ) 112 a b ab Bất đẳng thức luôn đúng và ta có đpcm. Bài tập tương tự Chứng minh rằng với mọi số thực a,b không âm thoả mãn 1 , 1; 2 ab a b < +≥ ta có 2 12 12 1 4 ( )( ) + +   + + a b a b a b ab . ≥   − − −−   ( )( ) 11 2 2) Kỹ thuật phân tích hằng đẳng thức n Phân tích thành tổng các bình phương ( )2 ∑ − ≥ x y . i i i 0 = 1 Ví dụ 1. Cho a,b,c là các số thực bất kỳ chứng minh a) 222 a b c ab bc ca ++≥++ . b) ( ) ( ) 2 ab bc ca abc a b c + + ≥ ++ 3 . c) ( ) (( ) ( ) ( ) ) ( ) 2 222 1 3 a b c b c c a a b ab bc ca ++ − − + − + − ≥ + + . 4 d) ( )( )( ) ( ) 222 2 a b c abc + + + ≥ ++ 2 2 23 . Lời giải a) Bất đẳng thức đã cho tương đương với: 222 2222220 a b c ab bc ca ++−−−≥ 2 22 22 2 ( ) ( ) ( ) . ⇔ − ++−++− +≥ a ab b b bc c c ca a 2 2 20 222 ( ) ( ) ( ) ⇔ − +− +− ≥ ab bc ca 0 Bất đẳng thức cuối đúng. Đẳng thức xảy ra khi và chỉ khi abc = = . 8 Cty TNHH MTV DVVH Khang Việt b) Thực hiện tương tự câu a) đưa về bất đẳng thức luôn đúng ( ) ( ) ( ) 222 ab bc bc ca ca ab − +− +− ≥ 0 . c) Ta có: ( ) ( ) ( ) 1 11 22 2 2 a b c b c a b c ab bc ca ab bc ca ++ − − = −− + + + ≥ + + . 3 4 12 Tương tự ta có: 1 1 2 2 ( ) ( ) a b c c a ab bc ca ++ − − ≥ + + 3 4 . 1 1 2 2 ( ) ( ) a b c a b ab bc ca ++ − − ≥ + + 3 4 Cộng lại theo vế ba bất đẳng thức trên ta có đpcm. Đẳng thức xảy ra khi và chỉ khi abc = = . d) Chú ý đẳng thức: 222 2 ( )( )( ) ( ) + + + − ++ 2 2 23 a b c abc 1 3 2 21 2 2 22 2 = + − + − + +−     ( ) ( ) ( ) ( ) . c a b ab ac bc 2 2 222 2 ( )( )( ) ( ) 2 2 23 ⇒ + + + ≥ ++ a b c abc Ta có điều phải chứng minh. Đẳng thức xảy ra khi và chỉ khi abc = = =1. Ví dụ 2. Cho xyz , , là các số thực thỏa mãn điều kiện 2 22 xyz ++=1. Chứng minh rằng a) 1 1 −≤ + + ≤ xy yz zx ; 2 8 2 3. 2 b) ( ) ( ) xy yz xz + + − ≥− 2 x y z xy yz ++ − − + 2 Lời giải a) Bất đẳng thức vế trái tương đương với: ( ) ( ) ( ) 2 22 2 2 10 2 0 0 xy yz zx xy yz zx x y z x y z + + +≥ ⇔ + + + + + ≥ ⇔ + + ≥ . Bất đẳng thức được chứng minh.  ++=  ++= Đẳng thức xảy ra khi và chỉ khi 2 220 ,1 chẳng hạn tại 1 1 ,2 2 x y =− = . Bất đẳng thức vế phải: 2 22 xyz xyz ( ) ( ) 1 − ++ =+ +− ++ xy yz zx x y z xy yz zx 1 0 222 (( ) ( ) ( ) ) = − +− +− ≥ xy yz zx 2 ⇒++≤ xy yz zx 1 9 Khám phá tư duy Kỹ thuật giải bất ĐT Bài toán Max – Min – Đặng Thành Nam b) Chú ý điều kiện ta rút gọn vế trái và đưa về chứng minh 8 2 3 ( ) 2 xy yz xzxy yz zx + + − ≥− ++ + 2 3 . 3 ( ) xy yz zx ++ + 2 10 ⇔ ≥ xy yz zx ++ + 2 3 Vậy ta chỉ cần chứng minh 2 22 + + ≥− =− − − 2 1 xy yz zx x y z 2 1 3 0 0   ⇔ + + + + ≥⇔ ++ + ≥     2 2 2 ( ) ( ) x z y yx z x z y y 2 4 Bất đẳng thức cuối đúng và ta có đpcm.  =  + + = ⇒ = = =− y Đẳng thức xảy ra khi và chỉ khi 0 1 11 0 , 0, 2 2 2 . xz y x y z  ++= 2 22 xyz 1 Ví dụ 3. Cho x,y,z là các số thực không âm. Chứng minh: a) 3 33 x y z xyz ++≥ 3 . 3 33 3 + +≥+ − − − xyz xyz x y y z z x . b) ( )( )( ) 3 4   + 3 c) y z x y z xyz x . 3 33 3 22 ++− ≥ −     Lời giải a) Bất đẳng thức đã cho tương đương với: 2 22 ( )( ) x y z x y z xy yz zx ++ + + − − − ≥ 0 . 1 0 222 ⇔ ++ − + − + − ≥     ( ) ( ) ( ) ( ) xyz xy yz zx 2 Bất đẳng thức được chứng minh. Đẳng thức xảy ra khi và chỉ khi xyz = = . b) Bất đẳng thức đã cho tương đương với: 1 3 222   ++ − + − + − ≥ − − −   ( ) ( ) ( ) ( ) ( )( )( ) xyz xy yz zx xyyzzx 6 4 . 222 ⇔ +++++ − +− +− ≥ − − −     ( ) ( ) ( ) ( ) ( ) ( ) ( ) ( )( )( ) xy yz zx xy yz zx xyyzzx 9 Chú ý x y x yy z y zz x z x +≥ − +≥ − +≥ − ; ; và sử dụng bất đẳng thức đã chứng minh được ở câu a) ta có 10 Cty TNHH MTV DVVH Khang Việt ( ) ( ) ( ) ( )( )( ) 3 xy yz zx xy yz zx xyyzzx + + + + + ≥−+ −+−≥ − − − 3 . 2 2 2 222 3 ( ) ( ) ( ) ( ) ( ) ( ) xy yz zx xy yz zx − +− +− ≥ − − − 3 Nhân theo vế hai bất đẳng thức trên ta có điều phải chứng minh. Đẳng thức xảy ra khi và chỉ khi xyz = = . +− ≤ y z x bất đẳng thức c) Theo câu a) ta có 3 33 x y z xyz ++− ≥ 3 0, do đó nếu 0 2 luôn đúng. + Ngược lại xét yz x yx zx +− >⇔ − + − > 20 0 ( ) ( ) . Đặt y a xz b x =+ =+ 2 ,2 bất đẳng thức trở thành ( ) ( )( ) 2 2 2 12 6 0. x a ab b a b a b −+ + + − ≥ += −> y z ab x . + Bất đẳng thức đúng vì 0 2 Đẳng thức xảy ra khi và chỉ khi abc = = hoặc b ca = = , 0 . Bài tập tương tự Cho a,b là hai số thực khác 0 thoả mãn điều kiện 1 1 ab ≥++ 3 a b .   1 1 3 Chứng minh rằng aba b . ≥ +     3 3 Ví dụ 4. Cho x,y,z là các số thực dương chứng minh 2 2 2 22 2 2 2 3( )( )( ) ( ) ( ) x xy y y yz z z zx x x y z xy yz zx + + + + + + ≥ ++ + + . Lời giải 31 3 22 22 22 2 Chú ý ( ) ( ) ( ) . x xy y x y x y x xy y x y ++ = + + − ⇒++ ≥ + 44 4 3 3 22 22 2 2 ( ) ( ) y yz z y z z zx x z x ++≥ + ++ ≥ + ; 4 4 Do đó ( ) ( ) ( ) 2 2 2 22 2 27 222 ( )( )( ) 64 x xy y y yz z z zx x x y y z z x ++ ++ ++ ≥ + + + . Ta chỉ cần chứng minh 64 ( )( )( ) ( )( ) 81 2 2 [ ] [ ] x y y z z x x y z xy yz zx + + + ≥ ++ + + ( )( )( ) 8 ( )( ) ⇔ + + + ≥ ++ + + x y y z z x x y z xy yz zx 9 222 ⇔ −+ −+− ≥ xy z yz x zx y ( ) ( ) ( )0 Bất đẳng thức cuối đúng. Ta có điều phải chứng minh. 11 Khám phá tư duy Kỹ thuật giải bất ĐT Bài toán Max – Min – Đặng Thành Nam Ví dụ 5. Cho a,b,c là các số thực dương thỏa mãn điều kiện 0 1 <≤≤≤ abc . Tìm giá trị lớn nhất của biểu thức P a bc a b c = + −+ ( ) ( ) . Lời giải Ta có ( ) ( ) ( ) P a b c a b c c ac bc a b ac bc a b = + − + = + −− ≤ + −− 2 2 1 1 11    ≤ + − − =− − − − + ≤       a bab a b 2 2 22 Đẳng thức xảy ra khi và chỉ khi 1 1, 2 c ab = = = . 3) Kỹ thuật thêm bớt hằng số Việc cộng hoặc trừ hai vế của bất đẳng thức cho một số nào đó làm lược bỏ đi phần phức tạp của bất đẳng thức. Ví dụ 1. Cho x,y,z là các số thực không âm thoả mãn điều kiện xyz ≥ ≥ . Chứng minh rằng xy yz zx x z ++ + a) 2 2 y yz z y z . ≥ + + + b) ( )( ) + + + + xy yz zx xzyz x xy y xz xzyz yz. ≥ ( ) ( )( ) ( ) 22 2 2 + + + ++ +++ Lời giải a) Bất đẳng thức đã cho tương đương với: 2 2 xy yz zx y yz z ++ ++ ≥ xz yz + + 2 2 xy yz zx y yz z y y ++ ++ . ⇔ −≥ − xz yz + + 22 0 0 zx zz x y z z x z z xy z ( ) ( ) ( ) ⇔ ≥ ⇔  + − + ≥ ⇔ − ≥   + + xz yz Bất đẳng thức cuối đúng. Đẳng thức xảy ra khi và chỉ khi xyz = = hoặc z xy = = 0, . b) Bất đẳng thức đã cho tương đương với: 2 2 ( ) ( )( ) ( ) ( )( ) xz xzyz yz xzyz + ++ +++ + + −≥ − 2 2 1 1 x xy y xy yz zx + + + + 22 2 ( ) ( )( ) ( ) zx y z zz x y z xy yz zx z x xy y + + 33 0 ⇔ ≥ ⇔ ++ + + − + + ≥   + +   + + 2 2 x xy y xy yz zx 12 Cty TNHH MTV DVVH Khang Việt Bất đẳng thức cuối đúng vì ( )( ) ( ) ( ) ( ) ( ) 2 2 2 3 33 + + + + ≥ +  + = + ≥ + +   x y z xy yz zx x y z x y z x y z x xy y . Bất đẳng thức được chứng minh. Đẳng thức xảy ra khi và chỉ khi z = 0 . 4) Kỹ thuật biến đổi với bất đẳng thức chứa căn + Phép bình phương hai vế được ưu tiên. + Cần chứng minh 1 2 12 + ++ ≥ + ++ ... ... A A A bb b n n . Ta có để chứng minh 22 2 A bc bb 1 11 1 1 = +≥ = . Rồi cộng lại theo vế các bất đẳng thức trên ta có đpcm. Ví dụ 1. Chứng minh với mọi số thực x,y cùng dấu và số thực k, ta có 22 2 k x k yk k xy ++ + ≥+ ++ . Lời giải Bất đẳng thức đã cho tương đương với 2 2 22 2 2 k x k y k xk y k x y kk x y ++ ++ + + ≥ +++ ++ 2 ( )( ) 2 2 . 42 2 k k x y xy k k x y xy ⇔ + + + ≥ ++ ⇔ ≥ () 0 Bất đẳng thức cuối đúng ta có đpcm. Đẳng thức xảy ra khi và chỉ khi hoặc x bằng 0 hoặc y bằng 0. Bài 2. Chứng minh rằng với x,y là hai số thực không âm thỏa mãn x y + ≥1, ta luôn có ( ) 2 2 2 x x y y xy xy +++ ++≤+ + +++ 4 42 4 . Lời giải Bất đẳng thức đã cho tương đương với ( )( ) 22 2 2 x y xy x x y y + ++ ++ ++ + + ≤ 82 4 4 ( ) ( ) 2 2 44 4 4 + + ++++ + +++ xy xy xy xy . ( )( ) ( ) 2 2 2 ⇔ ++ ++ ≤ + + +++ x x y y xy x y x y 4 42 4 . ( )( ) ( ) ( ) 2 2 22 2 2 4 4 4 44 4 ⇔ ++ ++ ≤ + + ++++ + +++     x x y y x y xy x y x y x y x y . ( )2 4 4 70 ⇔ + +++++− ≥       xy x y x y x y (luôn đúng do x y + ≥1). Tổng quát. Tương tự ta có các bất đẳng thức cùng dạng sau + Với mọi số thực không âm x,y ta luôn có ( ) 2 22 2 2 2 x xk y yk k xy xyk ++ + ++ ≤+ + +++ . 13 Khám phá tư duy Kỹ thuật giải bất ĐT Bài toán Max – Min – Đặng Thành Nam Đẳng thức xảy ra khi và chỉ khi x = 0 hoặc y = 0 . + Với mọi số thực không âm ta luôn có ( ) 2 2 2 x x y y xy xy − ++ − +≤+ + − − + 1 11 1 . Đẳng thức xảy ra khi và chỉ khi x = 0 hoặc y = 0 . + 1 1 1 1 , 0; , 1; 1 + + + ≥ + + + ≥ ≥− + ≥− a b a b ab a b a b ( ) . 5) Kỹ thuật đánh giá phân thức Sử dụng đánh giá cơ bản: 1 1 A B > >⇒ < 0A B . Ví dụ 1. Chứng minh rằng với mọi a,b,c dương ta có abc ab bc ac . 1 2 <++< +++ Lời giải Ta có : 1 1 a a ababcab abc ab abc +<++⇒ > ⇒ > (1) + ++ + ++ b b Tương tự ta có : > (2) c c bc abc , > (3) + ++ ac abc + ++ Cộng vế theo vế các bất đẳng thức (1), (2), (3), ta được : abc ab bc ac (*) ++>1 +++ Ta có : (4) + a ac aabab abc <+⇒ < + ++ bc abc , (6) + Tương tự : (5) + b ab < + ++ c cb < ca abc + ++ Cộng vế theo vế các bất đẳng thức (4), (5), (6), ta được : abc ab bc ac (**) ++< 2 +++ abc Từ (*) và (**) , ta được : 1 2 <++< ab bc ac (đpcm) +++ Ví dụ 2. Cho a,b,c là các số thực không âm thoả mãn điều kiện abc bc ca ab . ++= 3 111 +++ Chứng minh rằng 3 abc a bc b ca c ab . ++≥ 111 4 ++ ++ ++ Lời giải abc x y z xyz bc ca ab . Đặt ;; 3 111 = = = ⇒++= +++ 14 Cty TNHH MTV DVVH Khang Việt Ta cần chứng minh 3 xyz xyz . ++≥ 1114 +++ x x Chú ý 1 1 . ≥ x xyz + +++ y y ≥ 1 1 + +++ y xyz z z ≥ 1 1 + +++ z xyz Cộng theo vế ba bất đẳng thức trên ta có điều phải chứng minh. Đẳng thức xảy ra khi và chỉ khi có một số bằng 3 và hai số bằng 0. Ví dụ 3. Cho a,b,c là các số thực không âm có tổng bằng 1. Chứng minh 222 1117 +++ abc bca . ++≤ 222 111 2 +++ Lời giải Ta thấy dấu bằng đạt tại khi một số bằng 1 và hai số bằng 0. Vậy giả sử a abc = max , , . { } Khi đó ta mạnh dạn đánh giá 2 2 1 1;1 1 +≥+≥ b c . 2 1 1 1 1; a b a + Ta có 2 2 . + ≥⇒ ≤+ 1 + b 2 2 1 11 1 b c b + 2 2 + ≥⇒ ≤+ 1 + c 2 2 Suy ra c P ab abc 1 1 2 2 + 22 222 . ≤+ + + ≤+ + + + 2 2 1 1 + + a a 1 1 2 21 2 2 2 2 ( ) ( ) ≤+ + + + =+ + − + a bc a a 2 2 1 1 + + a a 1 7 2 11 2 2 2 Ta chỉ cần chứng minh ( ) . + +− + ≤ a aa 2 + 3 ( )( ) ⇔ − + −≤ a aa 14 3 1 0 Bất đẳng thức cuối luôn đúng nên ta có đpcm. Đẳng thức xảy ra khi và chỉ khi a bc = = = 1, 0 hoặc các hoán vị. kkk Chú ý. Bằng cách tương tự ta chứng minh được 1117 +++ abc bca . ++≤ kkk 111 2 +++ Với k là số nguyên dương. 15 Khám phá tư duy Kỹ thuật giải bất ĐT Bài toán Max – Min – Đặng Thành Nam Ví dụ 4. Cho x,y,z là các số thực thoả mãn điều kiện xy x y z , 1; 3 ≥− + + = . 2 2 x y Px y xy z z . − = + 1 Tìm giá trị lớn nhất của biểu thức ( ) 22 2 ++ + −+ 4 1 45 Lời giải Trước hết đánh gia hai mẫu số ở hai phân thức bằng cách thay z xy =−− 3 . 22 2 Ta chứng minh ( ) . 4 1 45 x y xy z z + + +≥ − + 2 2 2 ( ) ( ) 4 3 43 1 0 x y xy x y x y ⇔ + + − − − + − − −≥ ( ) ( )( ) 2 2 20 1 1 0 x y xy x y ⇔ + + +≥⇔ + + ≥ Bất đẳng thức đúng. Vậy ta có ( )2 2 2 + − +−− x y x y xy Pzz zz . 1 2 1 ≤ = 2 2 −+ −+ 45 45 Chú ý. xy x y z x y z ≥− − − = − + = − 1 4; 3 . Khi đó ( ) ( ) ( ) 2 2 2 zz z z z Pzz zz zz . 3 2 41 2 3 8 16 5 5 − − −− − − + ≤ = =− + ≤ 2 22 −+ −+ −+ 45 45 45 3 32 5 3 2 1, , 5 2 2 32 5 3   + = =     =− = =      + + = ⇔ =− ⇔  x y zx yz Dấu bằng đạt tại . x y z xy    = − = =− =     =  4 , 1, 3 2 2 xy z xy z z 2 6) Kỹ thuật đánh giá bất đẳng thức có chứa dấu giá trị tuyệt đối Sử dụng hai bất đẳng thức quen thuộc: x y x yx y x y + ≥+ − ≤− ; . Chú ý. Tư duy đầu tiên là khử dấu giá trị tuyệt đối muốn vậy ta xét trường hợp. Ví dụ 1. Chứng minh với mọi số thực a,b,c ta có a b c abc ab bc ca + + + ++ ≥ + + + + + . Lời giải Trong ba số a,b,c có ít nhất hai số cùng dấu không mất tính tổng quát giả sử là a và b khi đó a b ab + =+ . Vậy ta chỉ cần chứng minh c abc bc ca + ++ ≥ + + + 2 2 22 c a b c cc a b a c b c a c b c 2 2 ( ) ( ) ( ) ( ) ( )( ) ⇔ + ++ + ++ ≥ + + + + + + ( ) ( )( ) ( ) ⇔ + ++ ≥ + + = ++ + ab c c a b a c b c c c a b ab Bất đẳng thức cuối luôn đúng (đpcm). 16 Cty TNHH MTV DVVH Khang Việt Ví dụ 2. Cho x,y,z là các số thực đôi một không đồng thời bằng 0. Chứng minh 2 2 2222 ( )( )( ) 1 1 − −− −≤ ≤ xyyzzx xyyzzx . 2 2 222 2 ( )( )( ) + ++ Lời giải Ta có ( ) ( ) 2 2 2 2 2 2 2 2 2 2 22 x y x y x y x y xy −≤+⇔ − ≤ + ⇔ ≥ 4 0 . 2 2 2222 Từ đó suy ra ( )( )( ) xyyzzx − −− ≤ 1 2 2 222 2 ( )( )( ) xyyzzx + ++ 2 2 2222 ( )( )( ) − −− ⇔− ≤ ≤ xyyzzx 1 1 2 2 222 2 ( )( )( ) xyyzzx + ++ Bất đẳng thức được chứng minh. Ví dụ 3. Cho a,b,c là các số thực không âm chứng minh 3 3 . abc a b b c c a a b c + − + − + − ≥++ Lời giải Không mất tính tổng quát giả sử abc ≥ ≥ .Khi đó bất đẳng thức tương đương với: 3 3 ( )( )( ) abc a b b c c a a b c + − + − + − ≥++ 3 33 3 2 ( ) ⇔−− + ≥⇔ − + − ≥ a b c abc a b c ab c 3 3 0 ( )3 0 Bất đẳng thức cuối luôn đúng và ta có đpcm. Bài tập tương tự Cho a,b,c là các số thực dương thoả mãn điều kiện 3 3 1 abc a b b c c a +−+−+− = . Chứng minh rằng 13 a bc b ca c ab ++≤ . Ví dụ 4. Cho x,y,z là các số thực chứng minh 2 22 x y y z z x x y z xy yz zx −+ −+−≥ + + − − − 2 . Lời giải Không mất tính tổng quát giả sử xyz ≥ ≥ . Bất đẳng thức trở thành 111 ( )( )( )2 222 222 ( ) ( ) ( ) xy yz xz xy yz zx −+ −+−≥ − + − + − 222 ⇔ −≥ − +− +−     ( ) ( ) ( ) ( ) 2 2 xz xy yz zx 17 Khám phá tư duy Kỹ thuật giải bất ĐT Bài toán Max – Min – Đặng Thành Nam ( ) ( ) ( ) ( ) 2 222 4 2 ⇔ − ≥ − +− +−     xz xy yz zx 222 ( ) ( ) ( ) ⇔− ≥− +− xz xy yz 2 2 2 ( )( ) 2 0 [ ] ( ) ( ) ( )( ) ⇔ −+− ≥− +− ⇔ − −≥ xy yz xy yz xyyz Bất đẳng thức cuối luôn đúng ta có đpcm. Ví dụ 5. Cho n số thực 1 2 , ,..., n xx x (với n ≥ 3 ). Chứng minh + ++ − + − ++ − + − − ≥ + n n nn x x xx x x x x x x ... x ... , ,..., 2 { } 1 2 12 23 1 1 max x x xn n . 1 2 n Lời giải Chú ý. Với hai số thực x,y bất kỳ ta luôn có ++− = xy xy min , , , { } { } x y x y max x y ≤ ≤ và { } , 2 max x y . ++− = xy xy Sử dụng { } , 2 max x y ta được: x x xx x x x x x x + ++ − + − ++ − + − ... x ... n n nn 1 2 12 23 1 1 − + n n 2 xx xx xx xx xxxx ++− ++ − ++ − n n 12 12 23 23 1 1 ... 22 2 = + ++ nn n { } { } { } { } { } max x x max x x max x x max x xmax x x x , , ... , , , ,..., + ++ + nn n 12 23 1 1 − = ≤ n 1 2 n Bài toán được chứng minh. Dấu bằng đạt tại chẳng hạn 1 2 = = = ... xn x x . 7) Kỹ thuật đặt ẩn phụ Với bất đẳng thức đối xứng hai biến ta có thể đặt u a b v ab =+ = ; . Với phân thức ta có để đặt các mẫu số là các biến mới. Ví dụ 1. Chứng minh rằng với mọi a,b dương, ta có ( ) ( )( ) 22 2 2 a b a b a b ab +−≥+ − 2 1 . Lời giải Đặt 2 a b u ab v v += = > 2 , , 0. Khi đó bất đẳng thức tương đương với: 22 2 2 ( 2) ( )( 1) a b a b a b ab + −≥+ 2 2 − ⇔ + − −≥+ −   2 ( ) ( )( ) 2 2 1 a b a b ab a   42 2 2 (4 2 2) 2 ( 1) ⇔ − −≥ − v u v uv b ab 42 2 4 2 2 (1 ) ( ) 1 0 ⇔ − − − +≥ vu v u v v 18 Cty TNHH MTV DVVH Khang Việt Điều này chứng tỏ 2 2 2 82 1 ( 1) 8 ( 1) ≥v v vv −+ − + + uv . 4 4 Mặt khác 2+ = ≥= a b u ab v do đó ta chỉ cần chứng minh: 2 2 2 822 2 v v vv −+ − + + 1 ( 1) 8 ( 1) ( 1) ( 1)( 1) 0 v v v vv ≥ ⇔ − + ++ ≥ 4 v 4 Bất đẳng thức cuối đúng. Đẳng thức xảy ra khi và chỉ khi a b = =1. Ví dụ 2. Cho a,b,c là các số thực dương chứng minh rằng 22 22 2 2 xz yx zy 0 −−− ++≥ yz zx xy . +++ Lời giải Đặt a x yb y zc z x =+ =+ =+ , , khi đó vế trái của bất đẳng thức là ( ) ( ) ( ) a bc b ca c ab ab bc ca abc −−− + + = + + −−− b c a cab 222     111 0 ab bc bc ca ca ab = −+ −+ −≥     222 ca ab bc     Bất đẳng thức được chứng minh. Đẳng thức xảy ra khi và chỉ khi xyz = = . Ví dụ 3. Cho a,b,c là các số thực dương thỏa mãn điều kiện abc ++= 3. 444 abc ab bc ca bc ca ab . Chứng minh rằng ( ) ( ) ( ) 333 + + ≥++ +++ ++ + Lời giải Đặt  + =  + = abx bc y  + = caz do abc ,, 0 > và abc ++= 3 nên xyz ,, 0 > và  = −  = − 3 a y 3 b z  = − 3 c x . 4 4 43 3 3 −−− ++≥ + + yxz xyz yxz . Khi đó bất đẳng thức trở thành: 3 33 43 43 43 0 −−−       ⇔− +− +− ≥             xyz 333 xyz xyz ( )( ) ( )( ) ( )( ) 2 22 ⇔+ +≥ xx yy zz +− +− +− 12 12 120 xyz . 3 33 Bất đẳng thức cuối luôn đúng, từ đó ta có đpcm. 19 Khám phá tư duy Kỹ thuật giải bất ĐT Bài toán Max – Min – Đặng Thành Nam 8) Kỹ thuật sử dụng phép thế Từ bài toán có điều kiện từ hai biến trở lên ta rút một biến theo các biến còn lại rồi thay vào bất đẳng thức cần chứng minh. + Dạng này toán nếu có cần kết hợp đánh giá một số là max hoặc một số là min. Ví dụ 1. Cho a,b,c là các số thực không âm thoả mãn điều kiện ab bc ca ++=1 . Chứng minh rằng 5 2 +++ ≥ abc abc . 3 Lời giải Không mất tính tổng quát giả sử cba ≥ ≥ . 1 1 5. 2 Thay 1− = +ab ab ab a b ab − − ++ + ≥ ca b ta phải chứng minh ( ) 3 ab ab + + ( ) ( )2 25 3 1 0 ab ab a b ⇔ − + +− ≥ Bất đẳng thức hiển nhiên đúng vì 13 ab ≤ . 9) Kỹ thuật đánh giá theo cặp Áp dụng với dạng tích bất đẳng thức dạng tích. Ví dụ 1. Cho a,b,c là các số thực thuộc khoảng ( ) 0;1 . Chứng minh rằng ( )( )( ) ( )( )( ) 222 a a b b c c a bc b ca c ab − − − ≥− − − . Lời giải Không mất tính tổng quát giả sử abc ≥ ≥ .Khi đó do ( ) 0  − > a bc abcb ca . , , 0;1 0 ∈ ⇒  − > Nếu c ab − < 0 bất đẳng thức luôn đúng. Nếu c ab − ≥ 0 khi đó ta chứng minh bc a b ac c ab ( ) ( )( ) 1−≥ − − . Thật vậy ( ) ( )( ) ( ) ( )( ) 2 bc a b ac c ab bc a b ac c ab 1 1 −≥ − − ⇔ − ≥− − . ( ) ( ) 2 2 22 2 ⇔ − +≥ − − + ⇔ − ≥ bc a a bc ab ac a bc a b c 21 0 (luôn đúng).  −≥ − −  −≥ − − ac b a bc c ab Tương tự ta có: ( ) ( )( ) 1 . ( ) ( )( ) ab c a bc b ca 1 Nhân theo vế 3 bất đẳng thức trên ta có điều phải chứng minh. Đẳng thức xảy ra khi và chỉ khi abc = = . Ví dụ 2. Cho a,b,c là các số thực dương thỏa mãn điều kiện abc ++=1. Chứng minh rằng ( )( )( ) 222 8a b c a bc b ca c ab ≥− − − . 20 Cty TNHH MTV DVVH Khang Việt Lời giải Không mất tính tổng quát giả sử abc ≥ ≥ khi đó do ( ) 0  − > a bc abcb ca . , , 0;1 0 ∈ ⇒  − > Nếu c ab − < 0 bất đẳng thức luôn đúng. Nếu c ab − ≥ 0 khi đó ta chứng minh 2ab a bc b ca ≥− − ( )( ) . Thật vậy ( )( ) ( )( ) 2 2 2 4 ab a bc b ca a b a bc b ca ≥ − − ⇔ ≥− − . 22 2 2 2 22 2 2 2 ( ) ( ) ⇔ ≥ − − + ⇔ + + − +≥ 4 4 10 a b ab a c b c abc a b c a b ab c 22 2 22 2 22 ( ) ( ) ( ) ( ) ⇔ + − + − − ≥⇔ + − − − ≥ 4 2 10 4 1 0 a b c a b ab c c a b c a b ab c ( ) ( ) ( )( ) 22 2 4 00 ⇔ − + + − ≥⇔ − − ≥     ab ab a b c a b c ab a b (luôn đúng). Tương tự ta có 2 ;2 bc b ca c ab ca c ab a bc ≥− − ≥− − ( )( ) ( )( ) . Nhân theo vế 3 bất đẳng thức trên ta có ngay điều phải chứng minh. Đẳng thức xảy ra khi và chỉ khi 13 abc = = = . 10). Kỹ thuật sử dụng tính thuần nhất Đưa bất đẳng thức về dạng đồng bậc sẽ dễ xử lý hơn(xem thêm chương 3). Ví dụ 1. Cho a,b,c là các số thực thoả mãn điều kiện 222 abc ++= 3 . Chứng minh rằng ( ) ( ) ( ) 333 abc bca cab ++ ++ +≤ 6 . Lời giải Bất đẳng thức đã cho tương đương với: 2 3 3 3 222 2 ( ) ( ) ( ) ( ) abc bca cab a b c ++ ++ +≤ + + 3 4 4 4 22 22 22 2 2 2 2 2 2 ( ) ( ) ( ) ( ) ( ) ⇔ ++ + + + ≥ + + + + + 2 4 3 33 a b c a b b c c a ab a b bc b c ca c a Bất đẳng thức trên là tổng của ba bất đẳng thức có dạng: 4 4 22 2 2 4 2 ( ) ( ) ( ) a b a b ab a b a b ab a b ++ − + =− + − 4 3 2 2 2 ( ) ( ) =− −+ ≥ a b a ab b 0 Bất đẳng thức được chứng minh. Đẳng thức xảy ra khi và chỉ khi abc = = =1. 11) Biến đổi hàm lượng giác Ví dụ 1. Chứng minh với mọi số thực x ta có cos(sin ) sin(cos ) x x > . Lời giải   π Bất đẳng thức đã cho tương đương với: sin sin sin(cos ) 0 2x x   −− >   21 Khám phá tư duy Kỹ thuật giải bất ĐT Bài toán Max – Min – Đặng Thành Nam π π sin cos sin cos −+ −− xx xx 2 2 2cos .sin 0 ⇔ > . 2 2 Bất đẳng thức cuối luôn đúng do π π .   − = −≤ + = +≤     xx x xx x sin cos 2 sin 2; sin cos 2 sin 2 4 4 π π π x x − −− + 2 sin cos 2 π Vì vậy 22 2 0 ; 2 2 22 << ≤< π π π x x − −− + 2 sin cos 2 π 22 2 02 2 22 <≤ ≤< Bất đẳng thức được chứng minh. B. BÀI TOÁN CHỌN LỌC Bài 1. Cho xyz , , là các số thực dương thỏa mãn điều kiện xyz ≤ ≤ . Chứng minh rằng ( ) ( )   11 1 11   + + +≤ + +   y xz xz xz y xz . Lời giải BĐT tương đương với: ( ) ( ) 2 x z yx z x z + + + ≥ + xz xz y ( ) ( ) ( )( ) 2 2 ⇔ + ≥ + ⇔ − + + ≤⇔ − − ≤ y x z y zx y y z x zx y x y z 0 0 Bất đẳng thức cuối đúng vì 0 <≤≤ xyz . Ta có điều phải chứng minh. Đẳng thức xảy ra khi và chỉ khi xyz = = . Bài 2. Cho x,y,z là các số thực dương thoả mãn điều kiện xyz ≥ ≥ . Chứng minh 2 2 22 222 22 ( )( )( ) +++ xx y yz x zy z xyz xy zx yz . + + ≥++ +++ Lời giải Bất đẳng thức đã cho tương đương với: −       − +− − ≥ xy y x yz y z ( ) ( )1 1 1 1 0 xy xz xy yz xz + ++ ++    xy x y y z yz x y y z −− −− ⇔+≥ ( )( ) ( )( ) 0 x yx z x zy z ++ ++ ( )( ) ( )( ) Ta có điều phải chứng minh. 22 Cty TNHH MTV DVVH Khang Việt Bài 3. Cho xyz , , là các số thực thuộc đoạn [ ] 0;1 . Tìm giá trị lớn nhất của biểu thức ( ) ( ) 3 33 2 2 2 P x y z xy yz zx = ++ − + + 2 . Lời giải Ta có [ ] 32 3 2 32 xyz x x xy y yz z z , , 0;1 ; ; . ∈ ⇒≤≤ ≤≤ ≤≤ Từ đó suy ra ( ) 3 33 2 2 2 2 . x y z x xy yz z + + ≤ ++ ++ + ( ) 2 22 2 2 2 P x y z x y z xy yz zx ≤+++ + + − + + . Ta chứng minh ( ) 2 22 2 2 2 x y z x y z xy yz zx +++ + + − + + ≤ 3 . 2 22 ( ) ( ) ( ) ⇔ − + − + − ++ +−≤ x y y z z x xyz 1 1 1 30 2 22 x yy zz x . ( )( ) ( )( ) ( )( ) ⇔ − −+ − −+ − −≤ 11 11 11 0 Bất đẳng thức cuối đúng do ( )( ) ( )( ) ( )( ) 2 22 x yy zz x − −≤ − −≤ − −≤ 1 1 0; 1 1 0; 1 1 0 . Vậy giá trị lớn nhất của P bằng 3 xảy ra khi xyz = = =1. Bài 4. Chứng minh rằng với mọi số thực không âm a,b,c ta có ( ) ( ) 4 4 4 22 22 22 a b c abc a b c a b b c c a + + + ++ ≥ + + 2 . Lời giải Không mất tính tổng quát giả sử abc ≥ ≥ . Khi đó ( )( ) 2 ccacb − −≥ 0 và 2 2 22 − −+ − − = − −− −     ( )( ) ( )( ) ( ) ( ) ( ) a abac bbcba aba ac bbc 2 2 ≥ − −− −     ( ) ( ) ( ) ababc bbc 2 2 0 ( )( )( ) =− − − ≥ abbca b Bất đẳng thức được chứng minh. Đẳng thức xảy ra khi và chỉ khi abc = = . Nhận xét. Đây là một trường hợp riêng của bất đẳng thức Schur. Với a,b,c là các số thực không âm và k > 0 ta luôn có ( )( ) ( )( ) ( )( ) − −+ − −+ − −≥ 0 kkk a abac b bcba c cacb . Bài 5. Cho abc ,, 0 ≥ thỏa mãn điều kiện abc ++=1. Tìm giá trị lớn nhất của biểu thức ( ) ( ) ( ) 222 −− − =+ ++ ++ bc ca ab Pa b c . 444 Lời giải Chuyển mỗi biểu thức trong căn về cùng bậc hai ta có : 23 Khám phá tư duy Kỹ thuật giải bất ĐT Bài toán Max – Min – Đặng Thành Nam 2 2 ( ) ( ) ( ) bc bc − − + = ++ + a aa b c 4 4 ( ) ( ) 2 2 2 4 b c bc bc bc a a b c a bc a + −    + + = + ++ = + −≤ +      . 2 42 2 Suy ra ( )2 − + b c b c + ≤+ a a 4 2 Tương tự ta có : ( )2; 4 2 − + c a c a b b + ≤+ ( )2 − + a b a b + ≤+ c c 4 2 Cộng theo vế 3 bất đẳng thức trên ta được : 222 ( ) ( ) ( ) ( ) −− − = + + + + + ≤ ++ = bc ca ab P a b c abc 444 2 2. Vậy giá trị lớn nhất của P bằng 2 đạt tại ab c = = = 0, 1 hoặc các hoán vị. Nhận xét. Ta có thể tổng quát thành bài toán như sau : Cho a,b,c,k là các số thực không âm thỏa mãn abck ++= . Chứng minh rằng ( ) ( ) ( ) 2222 . −− − + ++ ++ ≤ bc ca ab ka kb kc k 444 Bài 6. Cho a,b,c là các số thực không âm thỏa mãn điều kiện abc ++=1. Chứng minh rằng ( ) ( ) ( ) 222 a bc b ca c ab +− + +− + +− ≥ 3. Lời giải Không mất tính tổng quát giả sử abc ≥ ≥ khi đó : Sử dụng bất đẳng thức Mincopsi ta có : ( ) ( ) ( ) 222 a bc b ca c ab +− + +− + +− ≥ ( ) ( ) ( ) ( ) 2 2 a b c ab bc ca + + + − + − + −    ( ) ( ) 2 2 = ++ + − a b c ac 4 . Bất đẳng thức được chứng minh nếu ta chứng minh được bất đẳng thức sau đúng. ( ) ( ) ( ) 2 2 a b c ac abc + + + − ≥ ++ 4 3 ( ) ( ) ( ) ( ) 222 2 ⇔ −≥ − + − + − 4 ac a b b c c a . 24 Cty TNHH MTV DVVH Khang Việt Ta có : 2 2 2   −+− =− +− + − − 2   ( ) ( ) ( ) ( ) ( )( ) ab bc ab bc abbc 2 2 ( ) ( ) ≥− +− ab bc 2 22 ( ) ( ) ( ) ⇒− ≥− +− ca ab bc Suy ra ( ) ( ) ( ) ( ) 222 2 ab bc ca ca − +− +− ≤ − 2 . Mặt khác : 2 22 2   −− − = − + −     ( ) ( ) ( ) ( ) 42 2 2 1 ac c a a c a c 2 ( ) ( ) = − + −+     2 2 14 a c a c ac ( ) 2 = − + + −+     2 2 14 a c a c c ac ( ) 2 ≥ − + + + −+     2 14 a c a b c c ac ( ) 2 =− + ≥     2 40 a c c ac Bài toán được chứng minh. Đằng thức xảy ra khi và chỉ khi 13 abc = = = hoặc a bc = = = 1, 0 và các hoán vị. Bài 7. Cho a,b,c là các số thực dương thỏa mãn điều kiện abc ++= 3 . Chứng minh rằng ( ) ( ) 22 22 22 2 2 2 2 33 ab bc ca a b c + + +≤ + + . Lời giải Không mất tính tổng quát giả sử a abc a = ⇒≤ min , , 1 { } . Ta có ( ) ( ) 22 22 22 2 2 2 2 33 0 ab bc ca a b c + + +− + + ≤ ( )( ) 2 2 2 22 2 ⇔ − + + +− ≤ 2 3 2 33 0 a b c bc a ( )( ) ( ) 2 22 2 2 2 ⇔ − + + + − +− ≤ 2 3 2 23 2 3 3 0 a b c b c a bc a ( )( ) ( ) 2 22 2 2 2 ⇔ = − − + + − +− ≤ P a a b c a bc a 2 3 3 2 23 2 3 3 0 . 2 2 3    + − bc a bc . Vì 2 Ta có ≤ =       2 2 a a ≤⇒ − > 1 32 0 do đó 25 Khám phá tư duy Kỹ thuật giải bất ĐT Bài toán Max – Min – Đặng Thành Nam 4 2 3 3 2 3 3 2. 2 3 2 3 3   − − ≤ − − + + − +−     a a Pa a a a 2 22 2 ( )( ) ( ) 2 2 . 3 1 3 14 1 0 2 2 ( ) ( ) = − − −≤ 8 a aa Vì ( ) ( ] 2 3 14 1 3 1 11 1 0, 0;1 a a aa a a − −= − − −< ∀∈ . Bài toán được chứng minh. Đẳng thức xảy ra khi và chỉ khi abc = = =1. Bài 8. Cho các số thực abc , , 0;1 ∈[ ] thỏa mãn 32 abc ++= . Tìm giá trị lớn nhất và giá trị nhỏ nhất của biểu thức ( ) 222 P abc = ++ cos . Lời giải ≤ + + ≤++= < a b c abc π Do abc , , 0;1 ∈[ ] nên 222 3 0 . 2 2 Vậy P lớn nhất( nhỏ nhất) khi 222 abc + + nhỏ nhất (lớn nhất) - Tìm giá trị nhỏ nhất của 222 abc + + . Ta có ( ) 222 1 3 2 a b c abc + + ≥ ++ = . Suy ra GTLN của P bằng 3 cos4 ; xảy ra khi 12 abc = = = 3 4 - Tìm giá trị lớn nhất của 222 abc + + . Giả sử 3 1 3 abc abc c c ≤≤⇒++= ≤ ⇒≥ . 2 2 2 222 2 22 2 2 3 5 22 4   ++=+ − +≤+ +=+ − ≤     a b c a b ab c a b c c c Vậy ( ) ( ) Do ( )( ) c c − −≤ 12 1 0 Suy ra GTNN của P bằng 5 cos4 ; xảy ra khi ( ) 1 , , 0,0, 2   =     abc hoặc các hoán vị. Bài 9. (TSĐH Khối D 2008) Cho x y, là các số thực không âm. Tìm giá trị lớn nhất, giá trị nhỏ nhất của biểu thức ( )( ) x y xy Px y. − − =+ + 1 ( ) ( ) 2 2 1 1 Lời giải Ta có : 2 2 2 2 ( ) ( ) x xy y yx x y xy x y y x x y P − − +− + + −+ = = = = − ( )( ) ( ) ( ) 1 11 . 22 22 22 2 2 ( ) ( ) ( ) ( ) ( ) ( ) ( ) ( ) ++ ++ ++ + + xy xy xy x y 11 11 11 1 1 26 Cty TNHH MTV DVVH Khang Việt 1 1 0 ;0 x y Với x y, 0 ≥ ta có : ( ) ( ) 2 2 . ≤ ≤≤ ≤ 1 1 4 4 x y + + Suy ra giá trị lớn nhất của P bằng 14 đạt tại x y = = 1, 0 và giá trị nhỏ nhất của P 14 − đạt tại x y = = 0, 1. Cách 2 : Ta có đánh giá thông qua trị tuyệt đối như sau : 2 ( )( ) x y xy x y xy x y xy P − − + ++ + + 1 1 1 1 ( )( ) ( ) . = ≤≤ = 22 2 22 2 1 1 1 1 41 1 4 ( ) ( ) ( ) ( ) ( ) ( ) xy xy xy ++ ++ ++ Do đó 1 1 −≤ ≤ P . Ta có kết quả tương tự. 4 4 Bài 10. Cho abc ,, 0 ≥ là các số đôi một khác nhau. Chứng minh rằng   111 4 ++ + + ≥   ( ) ( ) ( ) ( ) 222 . ab bc ca −−−   ab bc ca Lời giải Giả sử c abc = min , , { } , khi đó do abc ,, 0 ≥ ta suy ra: ab bc ca ab ++≥ ; 1 1 ≥ ; ( )2 2 b c − b 1 1 ≥ ( )2 2 a c − a     ++ ≥ 1 11 4 Vậy ta chỉ cần chứng minh : ( )222 aba b b a . −   ( ) 2 − ⇔ + + −≥⇔ + −≥ ab a b ab a b 40 20 2 2 ab ab b a ab ( ) ( ) − − 2   − ⇔− ≥   2 ( ) ab a b 2 a b ab ( ) 0 −   Bài toán được chứng minh. Xem thêm chương 3. Bài 11. Cho các số thực thoả mãn điều kiện abc ,, 0 > và 112 acb . + = ab cb Pab cb . + + Tìm giá trị nhỏ nhất của biểu thức2 2 = + − − 27 Khám phá tư duy Kỹ thuật giải bất ĐT Bài toán Max – Min – Đặng Thành Nam Lời giải Ta có 2 = +ac ba c thay vào biểu thức của P ta được : 2 2 ac ac a c + + ac ac a cc a ac Pac ac a c ca 333 1 4. 2 2 22 2 2 2 + + + +   = + = + =+ + ≥     − − + + a c ac ac (đúng theo AM-GM). Vậy giá trị nhỏ nhất của P bằng 4 đạt tại abc = = . Bài 12. Cho các số thực abc , , 1;3 ∈[ ] thỏa mãn điều kiện abc ++= 6 . Tìm giá trị lớn nhất của biểu thức 222 Pa b c =++ . Lời giải Đặt a x b y c z xyz =+ =+ =+ ∈ 1; 1; 1; , , 0;2 . [ ] Khi đó ( ) ( ) ( ) 222 2 22 Pa b c x y z = + + =+ ++ ++ 111 ( ) 2 22 = + + + ++ + x y z xyz 2 3 ( ) ( ) ( ) 2 = ++ − + + + ++ + x y z xy yz zx x y z 2 23 =− + + + 2 18 ( ) xy yz zx Từ xyz x y z , , 0;2 2 2 2 0 ∈ ⇒− − −≥ [ ] ( )( )( ) ⇔− ++ + + + − ≥ 84 2 0 ( ) ( ) x y z xy yz zx xyz ⇒− + + =− − ≤− 2 44 ( ) xy yz zx xyz do xyz ≥ 0 Từ đó suy ra P xy yz zx ≤− + + + ≤ 2 18 14 ( ) Đẳng thức xảy ra khi và chỉ khi ( ) ( ) abc , , 1,2,3 = hoặc các hoán vị Chú ý. Đặt ax by cz =+ =+ =+ 1; 1; 1 để chúng ta tận dụng tích xyz ≥ 0 Bài 13. Cho các số thực abc , , 0;1 ∈[ ]. Chứng minh rằng111 3 abc . ++≥ −−− abc 222 Lời giải Ta có : ( ) ( ) 2 1 10 2 12 a . − ≥ ⇒ − ≤⇒ ≥ − a aa a Tương tự ta có : 1 1 b c . ; 2 2 ≥ ≥ − − b c Cộng theo vế các bất đẳng thức trên ta được: 28 Cty TNHH MTV DVVH Khang Việt 111 3 3 3 abc do abc ≤1. + + ≥++≥ ≥ −−− a b c abc abc 222 Đẳng thức xảy ra khi và chỉ khi abc = = =1. Bài 14. Cho các số thực abc , , 0;1 ∈[ ] và abc ++≠ 0 . Chứng minh rằng 111 5 ab bc ca a b c + + + ++ . ++≤ 111 Lời giải Do bất đẳng thức đối xứng với ba biến nên không mất tính tổng quát ta giả sử 1 0 ≥≥≥≥ abc . Khi đó1 11 ( )( ) 2 + + +++ − − + c a b abc b c b c bc ++≤ ≤ = 111 1 1 ab bc ca bc bc +++ + + ab bc ca ab bc ca +++ + + +     + + = −+ −+ −+     +++ + + +     Mặt khác 1 1 13 ab bc ca ab bc ca 111 1 1 1 ( )( ) 11 11 11 ( )( ) ( )( ) 3 3 ab bc ca −− −− −− =− − − + ≤ ab bc ca +++ 111 Cộng theo vế các bất đẳng thức trên ta suy ra điều phải chứng minh. Đẳng thức xảy ra khi và chỉ khi ab c = = = 1, 0 hoặc các hoán vị. Bài tập tương tự Cho a,b,c là các số thực thuộc đoạn [0;1] và abc ++> 0. Tìm giá trị nhỏ nhất của biểu thức 5111 ab bc ca Pa b c ab bc ca . = +++ ++ + + + Bài 15. Cho các số thực không âm a,b,c. Chứng minh rằng ( )( )( )( ) 222 a b c abcabbcca + + ≥ ++ − − − 4 . Lời giải Không mất tính tổng quát ta giả sử b nằm giữa a và c , ta xét hai trường hợp - Nếu a b c VT VP ≥≥⇒ ≥≥ 0 , ta có đpcm. - Nếu cba ≥ ≥ , khi đó vế phải VP a b c a b b c c a = ++ − − − 4( )( )( )( ) = ++ − − − 4( )( )( )( ) abcbacbca ( )( ) ( )( ) 2 ≤ + + − + − −    abcba cbca Ta chỉ cần chứng minh ( )( ) ( )( ) 222 abcba cbca a b c ++ − + − − ≤ + + . Thật vậy bất đẳng thức này tương đương với : − +−≤ a a cb ( ) 22 0 , đúng và ta có đpcm. 29 Khám phá tư duy Kỹ thuật giải bất ĐT Bài toán Max – Min – Đặng Thành Nam C. BÀI TẬP RÈN LUYỆN Bài 1. Chứng minh rằng với mọi số thực không âm a và b ta có ( )( ) ( ) 2 2 a b a b ab a b ab ab + + ≥ +− 8 12 . Bài 2. Cho a,b,c là độ dài ba cạnh một tam giác thoả mãn abc ≤ ≤ . Chứng minh ( )2 a b c bc ++ ≤ 9 . 2 cos 2 cos 0; , ;2 cos 1  x ax a Bài 3. Cho ( ) − + π . Chứng minh rằng −≤ ≤ 1 1 y . a xyx xa ∈ ∈=− + 2 Bài 4. Cho x,y,z là các số thực dương thoả mãn điều kiện xyz ≥ ≥ . Chứng minh rằng 2 222 22 + + ≥++ xy yz zx xyz zxy . Bài 5. Cho a, b, c, d, e là các số thực. Chứng minh các bất đẳng thức 222 22 ( ) a b c d e ab c d e + + + + ≥ ++ + ; 222 2 ( ) a b c d ab c d + + + +≥ + + + 1 1 Bài 6. Cho a,b,c là các số thực dương. Chứng minh 333 a b c abc + + a ab b b bc c c ca a . ++≥ 2 22 22 2 3 ++ ++ ++ Bài 7. Cho x,y là hai số thực không âm không đồng thời bằng 0. Chứng minh 2 2 323 2 2 2 2 x xy y x y + +≥ + x y . 2 ( ) + 3 3 Bài 8. Cho x,y là 2 số thực dương. Chứng minh ( )( ) xyx y + + 9 18 . ≤ ≤ 2 2 2 ( ) x y + ab a b + + ab a b . Bài 9. Chứng minh với mọi số thực a và b ta có1 1 ≤ ++ + + Dấu bằng xảy ra khi nào? Bài 10. Cho a,b là các số thực và a khác 0. Chứng minh 2 221 + + +≥ 3 b a ba a . Bài 11. Chứng minh rằng với mọi số thực dương a,b,c ta có 222 + + ≥++ abc abc bca . Bài 12. Chứng minh rằng với mọi số thực a,b,c ta có ( ) 444 a b c abc a b c + + ≥ ++ . 30 Cty TNHH MTV DVVH Khang Việt Bài 13. Cho a,b,c là độ dài ba cạnh một tam giác. Chứng minh ++−−− <1 abcbca bcaabc . Bài 14. Cho a,b,c là độ dài ba cạnh một tam giác thoả mãn điều kiện abc < < . Chứng minh rằng ( ) ( ) ( ) 32 2 32 2 32 2 ab c bc a ca b −+ −+ −< 0 . Bài 15. Cho a,b,c là độ dài ba cạnh một tam giác. Chứng minh ( ) ( ) ( ) 222 333 ab c bc a ca b a b c − + − + + >++ . Bài 16. Cho a,b,c là độ dài ba cạnh một tam giác. Chứng minh 22 22 22 4 4 4 222 0 ab bc ca a b c + + −−−> . Bài 17. Cho x,y,z là các số thực dương có tích bằng 1 và z xyz = min , , . { } 2 22 2 . 1 ++ +     ≥ Chứng minh rằng xy z xy z . 2 12 1 2 1 ++ +   Bài 18. Cho a,b,c là các số thực không âm thoả mãn điều kiện ab bc ca ++=1 . Chứng minh rằng 1 1 152 ab bc ca +++ . ++≥ Bài 19. Chứng minh với mọi a,b,c là các số thực ta có ( ) ( ) ( ) 333 aa b bb c cc a ++ +++≥ 0 . Bài 20. Cho a,b,c là độ dài 3 cạnh một tam giác. Chứng minh abc a) ++< 2 bc ca ab ; ++ + b) ( ) 222 a b c ab bc ca ++< ++ 2 ; abc c) 3 . ++< 2 4 333 33 33 33 bc ac ba +++ 33 22 ab ab 3 Bài 21. Cho a,b là các số thực dương. Chứng minh + + a b . 2 ≤+ Bài 22. Chứng minh rằng với mọi số thực x, y thoả mãn điều kiện x y xy + ≥− ≤ 1, 2. Ta có 3 3 x y + ≥−7 . Bài 23. Cho a, b, c, d là các số thực. Chứng minh rằng ( )( ) ( ) 2 22 2 2 a b c d ac bd + + ≥+ . Bài 24. Cho a,b là 2 số thực không âm. Chứng minh rằng ta luôn có 2 2 3 3 11 2 2             ++ ++ ≥ + +       ab ba a b . 4 4 22 31 Khám phá tư duy Kỹ thuật giải bất ĐT Bài toán Max – Min – Đặng Thành Nam Bài 25. Cho xyz , , 0;2 ∈[ ] thỏa mãn xyz ++= 3. Tìm giá trị lớn nhất của biểu thức 2 22 Px y z =++ . Bài 26. Cho a,b,c là các số thực dương. Chứng minh 3 2 1 1 12   + + bc bc a a . a) + ≤+     333 abc b)( ) ( ) ( ) . 333 333 ++≥1 a bc b ca c ab ++ ++ ++ c) 2 a a bc abc . ≥ + ++ abc bc ca ab . d) ++≥ 2 ++ + 4 e) 3 a a ≥ . 444 2 ++ + 333 8 a bc abc abc f) 2221 . ++≥ a bc b ca c ab +++ 888 Bài 27. Cho a,b,c là các số thực đôi một phân biệt. Chứng minh 222 abc . 222 ++≥ 2 −− − ( ) ( ) ( ) bc ca ab Bài 28. Chứng minh rằng với ba số thực abc , , ta luôn có ( )( )( ) ( ) 222 2 a b c ab bc ca + + +≥ + + − 111 1 . Bài 29. Cho x,y,z là các số thực dương thỏa mãn x max x y z = { } , , . Chứng minh 111 x y z xyz + ++ y z x yzx . + + ≤++ +++ 111 Bài 30. Cho x,y là hai số thực dương thoả mãn điều kiện x y + = 2 1. 1 1 25 x y + xy . Chứng minh 2 + ≥ 1 48 Bài 31. Cho a,b là hai số thực khác 0 thoả mãn điều kiện 1 1 ab ≥++ 3 a b .   1 1 3 Chứng minh rằng aba b . ≥ +     3 3 32 Cty TNHH MTV DVVH Khang Việt Bài 32. Cho a,b,c,d là các số thực thuộc đoạn [ ] 0;2 . Chứng minh a b c d ab bc ca ad +++ ≤ + + + + + + + 1111 . Bài 33. Cho a, b, c là các số thực thuộc đoạn [ ] 0;2 và abc ++= 3 . 222 + + = + + Tìm giá trị lớn nhất và nhỏ nhất của biểu thức abc Pab bc ca . Bài 34. Cho a, b, c, d là các số thực thỏa mãn điều kiện 222 2 abcd +++ =1 . Tìm giá trị lớn nhất của biểu thức ( ) ( ) ( ) ( ) 333 3 Pabcd bcda cdab d abc = ++ + + + + ++ + ++ . Bài 35. Cho a, b, c là các số thực thuộc đoạn [ ] 1;3 thỏa mãn điều kiện 222 abc ++=14 . b c Chứng minh 12 8       − + ≥−    a a . Bài 36. Cho a,b,c là các số thực không âm. Chứng minh rằng ( ) 222 2 2 2 a b c ab bc ca a b c + + + + + +≥ ++ 9 5 . Bài 37. Cho a, b, c là các số thực dương. Chứng minh rằng ( ) ( )( ) 333 222 9 8 a b c abc a b c a b c + + + ≥ ++ + + . Bài 38. Cho a b, 0 > thỏa mãn điều kiện 2 2 a b + =1. Chứng minh rằng 2 1 1 2 2  a b ab b a . +≥ + −     Bài 39. Cho abc ,, 0 > thỏa mãn điều kiện 222 53 abc ++= . Chứng minh rằng111 1 a b c abc . +−< Bài 40. Cho a, b là hai số thực thỏa mãn a b, 1, < ta luôn có 112 + ≥ − − a b − ab . 2 2 1 1 1 Bài 41. Với mọi số thực a b, 1 ≥ ta luôn có 11 3 ab a b ab . ++ ≤ ++ + 1 1 12 Bài 42. Cho abc , , 0;1 ∈[ ]. Chứng minh rằng a bb cc a ( ) ( ) ( ) 1 1 1 1. −+ −+ −≤ Bài 43. Cho abc ,, 0 > thỏa mãn abc ≤ ≤ . Chứng minh rằng ( ) 111 1   −+ − + ≥     abcabc . 33 Khám phá tư duy Kỹ thuật giải bất ĐT Bài toán Max – Min – Đặng Thành Nam Bài 44. Cho các số thực abc , , thỏa mãn điều kiện abc ++=1. Tìm giá trị nhỏ nhất của biểu thức 2 22 22 2 P a ab b b bc c c ca a = ++ + +++ ++ . 2 22 2 Bài 45. Cho a,b,c,d thuộc đoạn [ ] 1;2 . Chứng minh rằng ( )( ) abcd + +≤ 25 . 2 12 ( ) ac bd + Bài 46. Cho x,y,z là các số thực. Chứng minh ( ) ( ) ( ) 222     − −− + +−−−≥   2 22 333 , , 444 xy yz zx x y z xy yz zx max .     Bài 47. Cho x,y,z là các số thực không âm. Chứng minh ( ) ( ) ( ) ( ) { } 3 33 3 22 2 3 ., 4 x y z xyz x y z max x y y z z x + + − ≥ ++ − − + − . Bài 48. Cho a,b,c là các số thực không âm. Chứng minh + +−≤ − − − abc abc a b b c c a { } ( ) ( ) ( ) 222 3 max ; ; 3 D. HƯỚNG DẪN GIẢI – ĐÁP SỐ Bài 1. Bất đẳng thức tương đương với: ( ) ( )( ) 2 26 0   − + + −≥     a b a b a b ab . Chú ý ( )( ) ( ) ( ) 2 2 a b a b ab a b ab a b ab + + − = − + +− ≥ 6 2 0. Bất đẳng thức được chứng minh. Đẳng thức xảy ra khi và chỉ khi a b = . Bài 2. Ta có ( ) ( ) 2 2 ab abc bc ≤⇒ ++ ≤ + 2 . Vậy ta chứng minh ( ) ( )( ) 2 2 2 2 9 45 0 4 0 b c bc b bc c b c b c + ≤ ⇔ − + ≤⇔ − − ≤ . Bất đẳng thức cuối luôn đúng do bc bc bac abc b −≤ −≥ +−= +− + > 0; 4 3 2 0 ( ) . Bất đẳng thức được chứng minh. Đẳng thức xảy ra khi và chỉ khi abc = = Bài tập tương tự Cho a,b,c là độ dài ba cạnh một tam giác ta có ( )2 22 2 2 2 4ab a b c > +− Bài 3. Ta có ( ) 2 2 x a − − = ≥ ⇒− ≤ ≤ 2 1 sin . 1 01 1 y y 2 2 ( ) − + 2 cos 1 x xa Bài 4. Bất đẳng thức tương đương với ( ) ( ) ( ) 2 220 −−− ++≥ xyz yzx zxy zxy . 34 Cty TNHH MTV DVVH Khang Việt Theo giả thiết ta có Suy ra 2 2  − −  ≥⇒ ≥  − − ≥≥⇒ ≥ xyz xyz ( ) ( ) y zz y . 2 2 ( ) ( ) yzx yzx xyzx y 2 2 2 2 22 ( ) ( ) ( ) ( ) ( ) ( ) xyz yzx zxy xyz yzx zxy − − − −+ −+ − ++≥ zxy y ( )( )( ) −−− = ≥ xyxzyz y 0 Ta có điều phải chứng minh. Đẳng thức xảy ra khi và chỉ khi xyz = = . Bài 5. Bất đẳng thức tương đương với 222 22 4444 44 ( ) a b c d e ab c d e + + + + ≥ ++ + 2 22 22 22 2 ( ) ( ) ( ) ( ) ⇔−+ +−+ +− + +−+ ≥ a ab b a ac c a ad d a ae e 44 44 4 4 44 0 22 22 ( ) ( ) ( ) ( ) ⇔− +− +− +− ≥ ab ac ad ae 2 2 2 2 0. Bất đẳng thức luôn đúng. Dẳng thức xảy ra khi và chỉ khi abcde = = = = 222 2 . Bất đẳng thứ hai là trường hợp riêng khi e =1 3 x xy − ≥ 2 Bài 6. Ta chứng minh x xy yvới mọi số thực dương x và y. 2 2 + + 3 Thật vậy bất đẳng thức tương đương với: ( )( )2 xyxy + −≥ 0 (luôn đúng). Áp dụng bất đẳng thức trên ta có 3 2; 3 − ≥ a ab 2 2 + + a ab b 3 2; 3 − ≥ b bc 2 2 + + b bc c 3 2 . 3 − ≥ c ca 2 2 + + c ca a Cộng theo vế ba bất đẳng thức trên ta có điều phải chứng minh. Đẳng thức xảy ra khi và chỉ khi abc = = . Bài tập tương tự Cho n là số nguyên dương lớn hơn hoặc bằng 2 và các số thực 1 2 , ,..., n xx x có tích bằng 1. Chứng minh 35 Khám phá tư duy Kỹ thuật giải bất ĐT Bài toán Max – Min – Đặng Thành Nam ( ) 9 9 + + ∑n i j x x n n + − ≥ 1 x xx x . 6 33 6 3 ≤< ≤ 1 i jn i ij j Bài 7. Bất đẳng thức đã cho tương đương với : 2 2 22 2   + +   ≥ + 323 2 2 x xy y x y 2 ( ) +   x y 2 2 2 22 2 4 ( ) ( )( ) ( ) 323 8 0 0 ⇔ + + − + + ≥⇔ − ≥ x xy y x y x y x y Bất đẳng thức được chứng minh. Đẳng thức xảy ra khi và chỉ khi x y = . 2 Bài 8. Bất đẳng thức vế trái tương đương với: ( ) xy x y . Dấu bằng xảy ra khi và chỉ khi x y = . 2 2 20 − ≥ ( ) x y + 2 2 2 Bất đẳng thức vế phải tương đương với: ( ) x xy y − +≥ 40 . 2 2 2 8 ( ) x y + Đẳng thức xảy ra khi và chỉ khi x y = ± ( ) 2 3 . Bài 9. Biến đổi tương đương bất đẳng thức đã cho về dạng luôn đúng a b ab + ≥+ . Đẳng thức xảy ra khi và chỉ khi ab ≥ 0 . Chú ý. Thực chất bất đẳng thức xuất phát từ tính đồng biến trên khoảng ( ) − +∞ 1; của hàm số1 = +x yx . Bài tập tương tự Chứng minh với mọi số thực a và b ta có2014 2014 ++ + + ab a b ab a b . ≤ ++ + + 2015 2015 2 1 3 3 0     + ++ − ≥   Bài 10. Bất đẳng thức tương đương với b a 2 2 4 a a 2 2 2 2 ( ) −   ⇔+ + ≥     2 3 1 0 a ba a 2 2 4 Bất đẳng thức được chứng minh. Đẳng thức xảy ra khi và chỉ khi 4 1 3 , 2 4 a . b a =− =± Bài 11. Bất đẳng thức chính là phần rút gọn của bất đẳng thức sau 36 Cty TNHH MTV DVVH Khang Việt ( ) ( ) ( ) 2220 −−− ++≥ ab bc ca bca . Đẳng thức xảy ra khi và chỉ khi abc = = . Bài 12. Bất đẳng thức đã cho chính là phần rút gọn của bất đẳng thức sau ( ) ( ) ( ) ( ) ( ) ( ) 222222 22 22 22 2 2 2 a b b c c a a bc b ca c ab − +− +− +− +− +− ≥ 0 . Đẳng thức xảy ra khi và chỉ khi abc = = . Bài 13. Chú ý đẳng thức 22 22 22 abcbcaa b b c c a −−− ++−−−= + + b c a a b c ab bc ca 22 22 22 ( ) ( ) ( ) −+ −+ − = ca b ab c ca b . abc ( )( )( ) −−− = acabcb abc Ta có ( )( )( ) 1 −−− < = acabcb abc abc abc . Bất đẳng thức được chứng minh. Bài 14. Chú ý nếu coi vế trái là một đa thức bậc ba của a thì ta có 2 hai nghiệm a ba c = = , . Vì vậy ta phân tích được vế trái dưới dạng ( )( )( )( ) a b a c b c ab bc ca − − − ++ . Rõ ràng với a b c a b a c b c ab bc ca <<⇒ − − − + + < ( )( )( )( ) 0 . Bất đẳng thức được chứng minh. Bài 15. Bất đẳng thức đã cho tương đương với 222 222 0     −− + −− + +− >     ( ) ( ) ( ) abc a bca b cab c ( )( )( ) ⇔ +− +− +− > abcbcacab 0 Bất đẳng thức cuối luôn đúng và ta có đpcm. Bài 16. Gọi P là biểu thức vế trái ta có đẳng thức sau P abcabcbcacab = ++ +− +− +− > ( )( )( )( ) 0 . Bất đẳng thức được chứng minh. Bài 17. Quy đồng rút gọn bất đẳng thức tương đương với : 2 2 12 2 2 2 ++≥ + + x y xz z z 2 ( ) ( ) ⇔− + − + ≥ 1 21 0 z z xy 37 Khám phá tư duy Kỹ thuật giải bất ĐT Bài toán Max – Min – Đặng Thành Nam Bất đẳng thức cuối đúng do vậy ta có đpcm. Đẳng thức xảy ra khi và chỉ khi xyz = = =1. Bài 18. Sử dụng bất đẳng thức 5 2 +++ ≥ abc abc và quy đồng rút gọn bất đẳng 3 thức cần chứng minh ( )( ) ( )( ) ( )( ) ( )( )( ) 2225 abac bcba cacb abbcca + ++ + ++ + +≥ + + + ( ) ( ) 2 ⇔ ++ + +≥ ++ 2 5 25 a b c abc a b c Sử dụng 5 63 abc a b c ≥− ++ ( )ta được 2 2 ( ) ( ) ( ) ( ) 2 5 25 2 88 a b c abc a b c a b c a b c ++ + +− ++ ≥ ++ +− ++ ( ) 2 = ++− ≥ 2 20 abc Bất đẳng thức được chứng minh. Đẳng thức xảy ra khi và chỉ khi ab c = = = 1, 0 hoặc các hoán vị.  =+−  =+ =+ =+⇒ = +−  =+− 2 axyz Bài 19. Đặt . x a by b cz c a b y z x ,, 2 2 Bất đẳng thức trở thành czxy ( ) ( ) ( ) 3 33 xxyz yyzx zzxy +− + +− + +− ≥ 0 . 4 443 3 3 3 3 3 ⇔+++ + + − − − ≥ x y z x y y z z x xy yz zx 0 Bài 20. a) Ta có 2 2 aa a = < + +++ ++ bc bcbc abc 2 2 bb b = < + +++ ++ ca caca abc 2 2 cc c = < + +++ ++ ab abab abc . Cộng theo vế 3 bất đẳng thức trên ta có điều phải chứng minh. b) Ta có 0 2 ( ) < <+⇒ < + a b c a ab c 0 2 ( ) . <<+⇒ < + b c a b bc a 0 2 ( ) <<+⇒ < + c a b c ca b Cộng theo vế ba bất đẳng thức trên ta có điều phải chứng minh. c) Ta có 38 Cty TNHH MTV DVVH Khang Việt 3 1 3 . ( ) ( ) ( ) ( ) ( ) ( ) 3 3 3 32 3 a b a b ab a b a b a b a b a b +=+ − +≥+ − + += + 4 4 . c c 3 ⇒ ≤+ + 3 3 3 4. a b a b Tương tự rồi cộng theo vế 3 bất đẳng thức trên ta có điều phải chứng minh. Bài 21. Bất đẳng thức đã cho tương đương với: 2 33 224 2 2 0 + +   ab ab a b a ab b ( ) ( ) a b . ≤ ⇔− ++ ≥   2 +   Bài 22. Bất đẳng thức đã cho tương đương với 3 3 ( ) x y xy xy + +− + + ≥ 13 3 2 0 2 2 ( )( ) ( ) . x y x y x y xy xy ⇔ + + + +− − − + + ≥ 1 1 3 20 1 1 1 1 3 20 222 ( ) ( ) ( ( ) ( ) ) ( ) x y x y x y xy ⇔ ++ − + − + − + + ≥ 2 Bất đẳng thức cuối luôn đúng theo giả thiết ta có đpcm. Bài 23. Ta có ( )( ) ( ) ( ) ( ) 2 22 2 222 a b c d ac bd ad bc ac bd + + =+ + − ≥+ . Đẳng thức xảy ra khi và chỉ khi ad bc = . 2 2 1 1 11 2 2 0; 0 ; 2 2 44 Bài 24. Ta có : Suy ra :   − ≥ − ≥⇒ + ≥ + ≥  a b a ab b . 2 3 3 11 1         ++ ++ ≥ ++ ++ = ++                 2 2 a b b a ab ab ab 4 4 22 2 . 2             = + ++ ≥ + + = + +                         1 1 11 1 1 4 22 a b ab a b 4 4 44 2 2 Bất đẳng thức được chứng minh. Đẳng thức xảy ra khi và chỉ khi 12 a b = = . Bài 25. Theo giả thiết ta có : ( ) ( ) ( ) ( ) 2 22 xx yy zz x y z x y z − + − + − ≤⇔ + + ≤ ++ = 2 2 20 2 6 . Bài 26. a) Đặt 0. + b c xa Ta cần chứng minh = > 1 1 1 24 1 2 3 22 3 ( ) ( ) + ≤+ ⇔ + ≥ + x xx x 2 432 2 2 ( ) ⇔ − + ≥⇔ − ≥ x x x xx 44 0 20 39 Khám phá tư duy Kỹ thuật giải bất ĐT Bài toán Max – Min – Đặng Thành Nam Bất đẳng thức cuối đúng suy ra đpcm. Đẳng thức xảy ra khi và chỉ khi bc a + = 2 . b)Áp dụng câu a) ta có 3 2 a a ; ≥ 3 3 222 + + + + a bc abc ( ) 3 2 b b ; ≥ 3 3 222 + + + + b ca abc ( ) 3 2 c c . ≥ 3 3 222 + + + + c ab abc ( ) Cộng theo vế ba bất đẳng thức trên ta có điều phải chứng minh. Đẳng thức xảy ra khi và chỉ khi abc = = . a) Bất đẳng thức tương đương với: 2 2 ( ) ( ) aa b c a b c ++ ≥ + 4 a a b c ab c aa b c . 2 2 ⇔ ++ − + ≥⇔ −− ≥     ( ) ( ) ( ) 40 0 Bất đẳng thức luôn đúng. Đẳng thức xảy ra khi và chỉ khi a = 0 hoặc abc = + . b) Áp dụng câu c) ta có a ab bc c 222 bc abc ca abc ab abc . ≥≥≥ ; ; + ++ + ++ + ++ Cộng theo vế ba bất đẳng thức trên ta có đpcm. Với a,b,c thì đẳng thức không xảy ra. c) Bất đẳng thức tương đương với: ( ) ( )( ) 2 4/3 4/3 4/3 8/3 4/3 4/3 4/3 4/3 4/3 abc a bc abc ++ −= + ++ 2 ≥ 2/3 2/3 1/3 2( ) .4 ( ) bc a bc ( ) ( ) 2 4/3 4/3 4/3 8/3 2/3 2/3 2 a b c a bc a a a bc ⇒ ++ ≥+ = + 8. 8 a a ⇒ ≥ 4/3 4/3 4/3 4/3 2 a bc abc + + + 8 Bất đẳng thức cuối đúng theo AM – GM. d) Áp dụng chứng minh ở câu e) xây dựng ba bất đẳng thức cùng dạng rồi cộng lại ta có điều phải chứng minh. Bài 27. Chú ý hằng đẳng thức bc ca ab abac bcba cacb . ( )( ) ( )( ) ( )( ) ++=1 −− −− −− 40 Cty TNHH MTV DVVH Khang Việt 222 2 a b c abc 222 2 2   + + = + + +≥   −− −   −− − Khi đó( ) ( ) ( ) bc ca ab bc ca ab . Bài 28. Chú ý đẳng thức : ( )( )( ) ( ) ( ) 222 2 2 a b c a b c ab bc ca ab bc ca + + + = ++− − − + + + − 111 1 . Bài 29. Bất đẳng thức tương đương với: xy yz xzxy yz ++ ++ 1 1 2 0 ( )( ) ( )( ) ( ) ( )( ) xy x y yz y z(luôn đúng) do − − +− ≥ ++ ++ 11 11 xz xy −≥ −≥ 0, 0 . Bất đẳng thức được chứng minh đẳng thức đạt tại xyz = = . Nhận xét. Nếu để tinh ý ta có thể khảo sát hàm ( ) +++ ta tb tc f t tb tc ta . Lúc này =++ +++ vế trái bất đẳng thức thay số 1 bởi một số dương bất kỳ bất đẳng thức vẫn đúng. Bài 30. Thay x y = −1 2 vào bất đẳng thức cần chứng minh đưa về chứng minh. 1 1 25 12 7 1 0 ( ) ( )2 2 y y y y(luôn đúng). +≥ ⇔ − + ≥ − + −y y 2 1 2 1 48 1 2 Bài toán được chứng minh. Đẳng thức xảy ra tại 1 1 ,2 4 x y = = hoặc 13 x y = = . 1 1 x ab y z xyz = =− =− ⇒ = ,, 1 Bài 31. Đặt 33 3 a b . Theo giả thiết ta có 3 33 x y z xyz + + ≥=3 3 ( )( ) 2 22 ⇔ ++ + + − − − ≥ x y z x y z xy yz zx 0 . Do ( ) ( ) ( ) 2 22 111 222 0 222 x y z xy yz zx x y y z z x + +−−−= − + − + − ≥ . 3 Do đó 1 1 0   + + ≥ ⇔ ≥− − ⇔ ≥ +     x y z x y z ab a b . 3 3 Bất đẳng thức được chứng minh. Bài 32. Xuất phát từ a b ab −≤∀ ∈ 2, , 0;2 [ ]ta được: ( )2 2 2 a b a b ab − ≤⇔ + ≤+ 4 42 . ( ) ( ) 2 ⇒ + ≤+ + = + ⇒+≤ + a b ab ab ab a b ab 4 2 2 41 2 1 . Tương tự ta có: b c bc c d cd d a da +≤ + + ≤ + +≤ + 21 ; 21 ; 21 . Cộng theo vế 4 bất đẳng thức trên ta có ngay điều phải chứng minh. 41 Khám phá tư duy Kỹ thuật giải bất ĐT Bài toán Max – Min – Đặng Thành Nam Bài 33. Ta có: ( ) ( ) 2 2 9 2 a b c ab bc ca ++ − + + = = − ++ ++ Pab bc ca ab bc ca . Sử dụng bất đẳng thức cơ bản ta có: ( ) 1 2 3 ab bc ca a b c + + ≤ ++ = suy ra 3 9 2 1 P ≥ −= . 3 Vậy giá trị nhỏ nhất của P bằng 1 đạt tại abc = = =1. Để tìm giá trị lớn nhất của P ta tìm giá trị nhỏ nhất của ab bc ca + + . Vì abc , , 0;2 ∈[ ]nên ( )( )( ) ( ) ( ) a b c abc ab bc ca a b c − − − ≤⇔ − + + + ++ −≤ 2 2 2 0 2 4 80 . 4 8 ( ) 4 95 2 2 + ++ − + abc a b c abc ab bc ca P . ⇔ + + ≥ = ≥⇒ ≤ −= 2 2 22 Vậy giá trị lớn nhất của P bằng 52đạt tại a bc = = = 2, 1, 0 và các hoán vị. Bài 34. Viết lại biểu thức P dưới dạng: ( ) ( ) ( ) ( ) ( ) ( ) 22 22 2 2 22 22 2 2 P ab a b bc b c cd c d da d a ac a c bd b d = ++ ++ + + ++ ++ + . Để ý: ( ) ( ) ( ) ( ) 4 4 22 4 − =++ − + ⇒ + = x y xy x y + + −− 4 4 4 22 2 2 2 2 6 x y x y x y xy x y xy x y . 6 44 Áp dụng vào bài toán suy ra 2 222 2 44 4 44 4 ( ) ( ) ( ) ( ) ( ) ( ) ( ) 3 +++ − +− +− + − +− +− = − abcd ab bc cd da ac bd P 4 4 . 2 222 2 ( ) abcd 3 3 +++ ≤ = 4 4 Vậy giá trị lớn nhất của P bằng 34đạt tại 12 abcd = = = = . Tổng quát. Cho n số thực không âm 1 2 , ,..., n xx x thỏa mãn mãn 1 2 + ++ = ... 1 n xx x .   =       n n ∑ ∑ Tìm giá trị lớn nhất của biểu thức 2 2 P xx . i j i jn j 1 1 ≤≠ ≤ = Bài 35. Đặt vế trái bất đẳng thức là P. Khi đó 42 Cty TNHH MTV DVVH Khang Việt 222 ( ) 2 2 2 2 14 2 2 2 a b c ac ab bc ab ac ab bc Pa a +++ − − − − − − = += + 2 2 2 ( ) 2 abc b b − + 7 7 2 2 = − − + ≥− − + 22 2 aa a a a 2 Mặt khác abc , , 1;3 ∈[ ] nên 27 − ≥− − ≥− ⇒ ≥− − + =− 3, 7 3 7 2 8 b P a a . Bất đẳng thức được chứng minh. Đẳng thức xảy ra khi và chỉ khi a = 1, b = 3, c = 2. Bài 36. Nhận xét. Dự đoán dấu bằng của bất đẳng thức đạt tại abc = = =1nên ta đặt ẩn phụ a x b y c z xyz = + = + = + ≥− 1, 1, 1, , , 1 ( ) đưa về chứng minh bất đẳng thức: ( ) ( ) ( ) ( )( ) ( )( ) ( )( ) 2 22 2 2 2 x y z xy yz zx + ++ ++ ++ + ++ + ++ + + 1 1 1 1 1 1 1 1 19 ≥ +++ 5 3 ( ) xyz . Khai triển và rút gọn ta được: ( ) ( ) 2 22 2 2 2 22 0 x y z xy yz zx xy yz zx + + + ++ + + + ≥ . ( ) ( ) ( ) ( ) 2 222 ⇔ ++ + + + + + + ≥ xyz xz yx zy 1 1 10 (luôn đúng). Bài toán được chứng minh. Đẳng thức xảy ra khi và chỉ khi xyz abc = ==⇔=== 0 1. Bài 37. Bất đẳng thức đã cho tương đương với: ( ) ( ) ( ) ( ) 333 5 94 + + + ≥ + + + + +    a b c abc ab a b bc b c ca c a . Theo bất đẳng thức AM-GM ta có: 333 a b c abc ++≥ 3 . Suy ra ( ) ( ) 333 333 5 94 3 a b c abc a b c abc ++ + ≥ +++ . Ta đi chứng minh: ( ) ( ) ( ) 333 a b c abc ab a b bc c a ca a b +++ ≥ ++ ++ + 3 . ⇔ − −+ − −+ − −≥ aa b a c bb c b a cc a c b ( )( ) ( )( ) ( )( ) 0 . Không mất tính tổng quát ta giả sử abc ≥ ≥ khi đó: cc a c b ( )( ) − −≥ 0 và ( )( ) ( )( ) ( ) ( ) ( ) aa b a c bb c b a a b aa c bb c − − + − − = −  − − −   ( ) ( ) ( ) ( ) 2 2 2 0 = − − + − = − +− ≥     a b a b cb a a b a b c Suy ra aa b a c bb c b a cc a c b ( )( ) ( )( ) ( )( ) − −+ − −+ − −≥ 0 . Bất đẳng thực được chứng minh. Đẳng thức xảy ra khi và chỉ khi abc = = . Bài 38. Bất đẳng thức cần chứng minh tương đương với 43 Khám phá tư duy Kỹ thuật giải bất ĐT Bài toán Max – Min – Đặng Thành Nam 1 1 + ≥ + + −⇔++ ≥ + 22 2 2 22 1 a b a b ab ab ab ba ( ) ( )2   + − 2 1 a b ⇔ + + + −≥ +   ab ab 1 22 1 2     ( ) 2 ⇔ − ++ − ≥ 1 2 2 2 0 (*) t t ; với = +∈(1; 2 tab (Vì ( )2 22 22 a b a b ab a b a b + = + + > + =⇒ + > 2 11 Và ( ) ( ) 2 2 2 ab a b ab + ≤ + =⇒+≤ 22 2 Suy ra ∈(1; 2 t ). Bất đẳng thức (*) luôn đúng với ∈(1; 2 t . Bài 39. Do abc ,, 0 > nên bất đẳng thức tương đương với : bc ca ab +−<1 Ta có : ( ) ( ) 2 1 5 222 0 1 a b c bc ca ab a b c +− ≥⇒ + − ≤ + + = < (luôn đúng). 2 6 Bài toán được chứng minh. 11 2 Bài 40. Ta có: + ≥ − − − − a b a b. ( )( ) 2 2 2 2 1 1 1 1 Ta chứng minh ( )( ) ( ) 2 2 2 2 22 22 2 1 1 1 1 12 0 − − ≤− ⇔− − + ≤− + ⇔ − ≥ a b ab a b a b ab a b a b (luôn đúng). Bài 41. Quy đồng rút gọn đưa về bất đẳng thức: ( )( )( ) ab a b − − −≥ 1 1 10 (luôn đúng do a b, 1 ≥ ). Bài 42. Bất đẳng thức cần chứng minh tương đương với : a b c ab bc ca ++− + + ≤ ( ) 1 Xuất phát từ giả thiết ta có : ( )( )( ) ( ) ( ) 1 1 1 01 0 − − − ≥ ⇔− + + + + + − ≥ a b c a b c ab bc ca abc ⇒ + + − + + ≤− ≤ ( ) ( ) a b c ab bc ca abc 1 1 Đẳng thức xảy ra khi và chỉ khi ab c = = = 0, 1hoặc các hoán vị. Bài 43. Bất đẳng thức cần chứng minh tương đương với 111 1 111 1 −+≥ ⇔+≥+ a b c abc a c b abc −+ −+ ac ac + + 1 1 ⇔ ≥ ⇔≥ ( ) ( ) ac b a b c ac b a b c −+ −+ 44 Cty TNHH MTV DVVH Khang Việt ⇔ −+ ≥ ⇔ − − − ≥⇔ − − ≥ b a b c ac a b c b b c a b b c ( ) ( ) ( ) ( )( ) 0 0 Bất đẳng thức cuối luôn đúng do abc ≤ ≤ . Ta có đpcm. Bài 44. Ta có ( ) ( ) ( ) 2 2 313 222 a ab b a b a b a b ++= + + − ≥ + . 444 ( ) 2 2 3 3 ⇒ + + ≥ +≥ + a ab b a b a b . 2 2 Tương tự ta có : ( ) 2 2 3 3 b bc c b c b c + + ≥ +≥ + ; 2 2 ( ) 2 2 3 3 c ac a c a c a + + ≥ +≥ + . 2 2 Cộng theo vế ba bất đẳng thức trên ta được : P abc ≥ ++ = 3 3 ( ) . Đẳng thức xảy ra khi và chỉ khi 13 abc = = = . Bài 46. Giả sử ( ) ( ) ( ) ( ) 2 222 3 333 , , 4 444 − − −−     =   xy xy yz zx max .     Ta chứng minh ( )2 2 22 3 − + +−−−≥ x y x y z xy yz zx . 4 2 22 22 ⇔ + +−−−≥+ − 44 4444336 x y z xy yz zx x y xy . ( ) 22 2 2 ⇔ + + + − − ≥⇔ +− ≥ x y z xy yz zx x y z 42440 2 0 (luôn đúng). Bài toán được chứng minh. Bài 47. Ta có: ( )( ) 3 33 2 22 x y z xyz x y z x y z xy yz zx + + − = ++ + + − − − 3 . Bài toán đưa về chứng minh ( ) ( ) ( ) 222     − −− + +−−−≥   2 22 333 , , 444 xy yz zx x y z xy yz zx max .     Đây chính là kết quả bài toán trên và ta có điều phải chứng minh. CHỦ ĐỀ 2: KỸ THUẬT CHỨNG MINH PHẢN CHỨNG A. NỘI DUNG PHƯƠNG PHÁP Giả sử cần chứng minh bất đẳng thức nào đó đúng, ta giả sử bất đẳng thức đó là sai và kết hợp với điều kiện giả thiết chỉ ra điều vô lý. Điều vô lý có thể là trái với giả thiết hoặc trái với một điều đúng. Từ đó suy ra bất đẳng thức cần chứng minh là đúng. 45 Khám phá tư duy Kỹ thuật giải bất ĐT Bài toán Max – Min – Đặng Thành Nam B. BÀI TẬP MẪU Bài 1. Chứng minh rằng ít nhất một trong các bất đẳng thức sau là đúng 22 22 22 a b bc b c ca c a ab +≥ +≥ +≥ 2; 2; 2 . Lời giải 2 2  + <  + < 2 a b bc Giả sử tất cả các bất đẳng thức trên đều sai khi đó 2 2 2 . b c ca + <  2 2 2 c a ab Cộng theo vế 3 bất đẳng thức trên ta được ( ) ( ) 222 2 2 a b c ab bc ca ++ < ++ ( ) ( ) ( ) 2 22 22 2 ⇔ − ++−++− +< a ab b b bc c c ca a 2 2 20 ( ) ( ) ( ) 222 ⇔ − +− +− < ab bc ca 0 (mâu thuẫn). Vì vậy điều phản chứng là sai nên khẳng định đề bài đúng (đpcm). Bài 2. Cho a,b,c là các số thực thuộc khoảng ( ) 0;1 .Chứng minh ít nhất một trong các bất đẳng thức sau là đúng ( ) ( ) ( ) 111 1 ;1 ;1 444 ab bc ca −≤ −≤ −≤ . Lời giải 1 14  − >  − > ( ) a b 1 14 Giả sử tất cả các bất đẳng thức trên đều sai khi đó ( ) b c − >  1 14 ( ) c a . Nhân theo vế ba bất đẳng thức trên ta được ( ) ( ) ( ) 1 11164 a ab bc c − − −> (1). 2 1 1 0 12 4 a a Chú ý. ( )   + − . < −≤ =     a a 2 1 1 0 12 4   + − b b b b ( ) < −≤ =     2 1 1 0 12 4   + − ( ) c c < −≤ =     c c Nhân theo vế ba bất đẳng thức trên ta được ( ) ( ) ( ) 1 111 , 64 a ab bc c − − −≤ dẫn tới mâu thuẫn với (1). 46 Cty TNHH MTV DVVH Khang Việt Vậy điều phản chứng là sai do đó ít nhất một trong các bất đẳng thức đã cho là đúng (đpcm). Bài tập tương tự Cho a,b,c là các số thực thuộc khoảng ( ) 0;1 .Chứng minh ít nhất một trong các bất đẳng thức sau là sai ( ) ( ) ( ) 111 1 ;1 ;1 444 ab bc ca −> −> −> . Bài 3. (HOMC 2007) Cho p abcd = là một số nguyên tố có bốn chữ số. Chứng minh rằng phương trình 3 2 ax bx cx d + + += 0 không có nghiệm hữu tỷ. Lời giải Giả sử phương trình có nghiệm hữu tỷ thì nghiệm này phải âm giả sử nghiệm đó là ( ) * 0 =− ∈ = , , , , 1.  p x pq pq qKhi đó 3 232 23 − + − + = ⇔− + − + = ⇒  . 3 2 0 0 ppp a q a b c d ap bp a cpq dq  q q q d p Do đó p,q là các số tự nhiên có một chữ số và vì p,q là nghiệm của phương trình nên ( )( ) 32 2 * f x ax bx cx d qx p ex fx g e f g () ,, , = + + += + + + ∈ . Ta có abcd f q p e f g qp efg = = + + += (10) 10 100 10 . ( )( ) trái với giả thiết abcd là một số nguyên tố. Vậy điều phản chứng là sai ta có điều phải chứng minh. Bài tập tương tự (HOMC 2007) Cho p abc = là một số nguyên tố có ba chữ số. Chứng minh rằng phương trình 2 ax bx c + += 0 không có nghiệm hữu tỷ. Bài 4. Giả sử 2 2 1 1 12 2 2 f x x ax b f x x a x b () ; () =+ + =+ + là hai tam thức bậc hai với hệ số nguyên có nghiệm chung là a. Chứng minh rằng nếu a không là số nguyên thì tam thức bậc hai sau luôn có nghiệm thực ( ) 212 12 fx x a a x b b ( ) = + + ++ . Lời giải Ta chứng minh a không thể là số hữu tỷ thật vậy giả sử ,, , , 1 ( ) p a pq pq =∈ =  . q Do a là nghiệm của 2 1 11 1 1 1 1 2 () 0 0 0 p p f x a a a b a b p a pq b q 2 22 ⇒ + + =⇔ + + =⇔ + + = . q q 47 Khám phá tư duy Kỹ thuật giải bất ĐT Bài toán Max – Min – Đặng Thành Nam Suy ra ( ) 2 p q pq pq q a   ⇒ ⇒ = =⇒ ∈ , 1 trái với giả thiết a không là số nguyên. Vì vậy a không là số hữu tỷ. Do a là nghiệm chung của 21 1  + += a aa b 0 ( ) ⇒ ⇒ − =−  + +=. ( ), ( ) (1) fxf x a a a b b 1 2 1 2 21 22 2 a aa b 0 Do a không là số hữu tỷ nên 1 21 2 (1) ; ⇔= = a ab b . Khi đó 2 2 1 11 1 1 f x x ax b f x x ax b () ; () 2 2 =+ + =+ + . Do 1f x( ) có nghiệm nên 21 1 a b − ≥ 4 0 khi đó f(x) có 1 1 '2 4 0 ( ) 222 ∆= − = − + ≥ ab ab a . 11 11 1 2 2 Điều đó chứng đó f(x) có nghiệm ta có đpcm. Bài 5. Cho a,b,c là các số thực dương thoả mãn điều kiện 111 3 ++= . abc Chứng minh rằng ab bc ca ++ ++ +≥ 3 2 . Lời giải 222  =+−  + ++  = = = ⇒=+− =+− . ax z y ab bc ca x y z bx y z Đặt , , 222 2 22 222 cz y x Ta cần chứng minh xyz ++≥ 3 với điều kiện 111 3 +− +− +− . ++= 2 22 222 22 2 xyz yzx zxy Thật vậy giả sử xyz ++< 0.Khi đó sử dụng bất đẳng thức AM – GM cho ba số dương ta có 111 3 +− +− +− +− +− +−. ++≥ ( )( )( ) 2 22 222 22 2 2 22 2222 2 2 3 xyz yzx zxy xyzyzxzxy Mặt khác 6 xyz x y z y z x z x y xyz  + + 2 2 2 2 2 2 2 2 2 2 22 1 ( )( )( ) +− +− +− ≤ ≤ <     . 3 111 3 Suy ra 2 22 222 22 2 +− +− +−mâu thuẫn với giả thiết. ++> xyz yzx zxy Vậy điều phản chứng là sai và ta có điều phải chứng minh. 48 Cty TNHH MTV DVVH Khang Việt Ta cùng xét bài toán quen thuộc sau trích từ đề thi IMO 2001 Bài 6.( IMO 2001) Cho a,b,c là các số thực dương. Chứng minh rằng: abc . 2221 ++≥ a bc b ac c ba +++ 888 Lời giải abc Đặt 2 22 ; ; ,( ; ; (0;1)) x y z xyz = = = ∈ a bc b ac c ba +++ 888 2 22 22 2 a xb yc z Để ý rằng: bc ac ba xyz = = = −−− . 222 ; ; 888 111 2 22 Suy ra 1 ( )( )( ) 512 111 xyz xyz = −−− . 2 22 Ta cần chứng minh xyz ++≥1 với xyz , , (0;1) ∈ và 2 22 2 (1 )(1 )(1 ) 512( ) − − −= x y z xyz (1). Giả sử ngược lại xyz ++<1. Theo bất đẳng thức AM – GM ta có: 2 2 2 22 2 2 22 (1 )(1 )(1 ) [( ) ][( ) ][( ) ] − − − > ++ − ++ − ++ − x y z xyz x xyz y xyz z . = +++ + +++ + +++ + ( )( )( )( )( )( ) x x y zy zx y z yz xz z x yx y 11 11 11 2 22 2 42 42 42 4( ) .2( ) .4( ) .2( ) .4( ) .2( ) 512( ) x yz yz y zx xz z xy xy xyz ≥ = Điều này mẫu thuẫn với (1). Vậy điều phản chứng là sai và ta có điều phải chứng minh. Bài tập tương tự Cho a,b,c là các số thực dương có tích bằng 1. Chứng minh 111 1 +++ . ++≤ 51 51 51 abc Bài 7. Cho a,b,c là các số thực dương thỏa điều kiện a b c abc +++ = 4 . Chứng minh rằng: a b c ab bc ca ++≥ + + . Lời giải Giả sử ngược lại có a b c ab bc ac ++< + + . Sử dụng bất đẳng thức Schur bậc 3 ta có(Xem chương 4). abc ab bc ac a b c 9 2 4( ) ( ) abc + + ≥ + + − ++ > ++ − ++ = ++ ( )[4 ( )] ( ) a b c a b c abc a b c ⇒++< abc 3 . Khi đó abc abc a b c <⇒ = + + + < 14 4 . Mâu thuẫn. 49 Khám phá tư duy Kỹ thuật giải bất ĐT Bài toán Max – Min – Đặng Thành Nam Suy ra bất đẳng thức đầu đúng. Từ bất đẳng thức này ta chứng minh được một bất đẳng thức khó sau Cho a,b,c là các số thực dương và k số thực thoả mãn điều kiện 2 4 2 10 k k + −≥ ta luôn có 3 12 abc kkk k             + + + ≥+       ++ + . bc ca ab C. BÀI TẬP RÈN LUYỆN Bài 1. Chứng minh rằng nếu a b cd + = 2 thì ít nhất một trong các bất đẳng thức sau là đúng 2 2 c ad b ≥ ≥ ; . Bài 2. Chứng minh rằng ít nhất một trong ba bất đẳng thức sau là đúng ( ) ( ) ( ) 222 bc ca ab ++ + 22 22 22 ; ; 222 +≥ +≥ +≥ . ab bc ca Bài 3. Cho a,b,c là các số thực thuộc khoảng ( ) 0;2 .Chứng minh rằng ít nhất một trong các bất đẳng thức sau là sai abbcca ( ) ( ) ( ) 2 1; 2 1; 2 1 −> −> −> . Bài 4. Cho a,b,c là các số thực thoả mãn a b c ab bc ca abc ++> + + > > 0; 0; 0 . Chứng minh rằng abc >>> 0, 0, 0 . Bài 5. Chứng minh rằng nếu aa b b 12 1 2 ≥ + 2( )thì ít nhất một trong hai phương trình sau có nghiệm 2 2 11 2 2 x ax b x a x b + += + + = 0; 0 . Bài 6. Cho abc ≠ 0. Chứng minh ít nhất một trong ba phương trình sau có nghiệm 222 ax bx c bx cx a cx ax b + += + += + += 2 0; 2 0; 2 0 . Bài 7. Cho 2 f x x ax b ( ) =++ . Chứng minh rằng với mọi giá trị của a và b thì ít nhất một trong ba số f ff (0) , (1) , ( 1) − lớn hơn hoặc bằng 12. Bài 8. Cho a,b,c là các số thực thoả mãn điều kiện 1. 222 abc a b c + + =− + + = 2; 2 . 2. Chứng minh rằng cả ba số a,b,c đều thuộc đoạn 4 ;0   −   . 3 Bài 9. Cho a,b,c là các số thực thoả mãn điều kiện 2 2 a b ab bc ca ++++< 0 . Chứng minh rằng 222 abc + < . Bài 10. Cho a,b,c là các số thực đôi một khác nhau chứng minh rằng ít nhất một trong ba bất đẳng thức sau là sai ( ) ( ) ( ) 222 9 ;9 ;9 ab a b c bc a b c ca a b c ≥ ++ ≥ ++ ≥ ++ . 50 Cty TNHH MTV DVVH Khang Việt Bài 11. Cho a,b,c,d là các số thực dương chứng minh ít nhất một trong các bất đẳng thức sau là sai a b c d a b c d ab cd a b cd c d ab +<+ + + < + + < + ; ; ( )( ) ( ) ( ) . Bài 12. Cho a,b,c là các số thực dương có tích bằng 1. Chứng minh rằng: 111 1 +++ ++≥ 18 18 18 abc Bài 13. Cho a,b,c,d là các số thực chứng minh. Chứng minh rằng: 22 2 2 1 min{ ; ; ; } 4 abbccd d a − − − −≤ Bài 14. (USAMO 2000 ) Cho a,b,c là các số thực dương. Chứng minh rằng: + +−≤ − − − abc abc max a b b c c a { } 3 222 ( ) ;( ) ;( ) 3 Bài 15. Cho a,b,c là các số thực dương và k số thực thoả mãn điều kiện 2 4 2 10 k k + −≥ ta luôn có 3 12 abc kkk k             + + + ≥+       ++ + . bc ca ab D. HƯỚNG DẪN GIẢI – ĐÁP SỐ Bài 1. Giả sử cả hai bất đẳng thức đều sai khi đó 2 c a < ; 2 d b < Cộng theo vế hai bất đẳng thức trên ta được ( ) 2 2 2 c d a b cd c d + <+= ⇔ − < 2 0 vô lý. Vậy điều phản chứng là sai ta có điều phải chứng minh. Bài tập tương tự Cho a,b,c,d là các số thực thoả mãn điều kiện ac b d ≥ + 2( ). Chứng minh rằng ít nhất một trong các bất đẳng thức sau là sai 2 2 a bc d < < 4; 4 . Bài 2. Giả sử tất cả các bất đẳng thức trên đều sai khi đó 2 2 2 a b ( ) b c + + < ; 2 2 2 2 b c ( ) c a + + < ; 2 2 2 2 c a ( ) a b + + < . 2 Cộng theo vế ba bất đẳng thức trên ta được 51 Khám phá tư duy Kỹ thuật giải bất ĐT Bài toán Max – Min – Đặng Thành Nam 222 ( ) ( ) ( ) ( ) ab bc ca 222 + ++ ++ 22 abc ++ < 222 ( ) ( ) ⇔ ++ < ++ 2 2 a b c ab bc ca 222 ( ) ( ) ( ) ⇔ − +− +− < (vô lý). ab bc ca 0 Điều phản chứng là sai do đó ta có điều phải chứng minh. Bài 3. Giả sử tất cả các bất đẳng thức trên là đúng khi đó abbcca ( ) ( ) ( ) 2 1; 2 1; 2 1 −> −> −> . Nhân theo vế ba bất đẳng thức trên ta được a ab bc c ( ) ( ) ( ) 2 2 21 − − −> (1). Chú ý. 2 ( ) ( ) 0 2 1 11 aa a < − =− − + ≤ 2 ( ) ( ) . 0 2 1 11 bb b < − =− − + ≤ 2 ( ) ( ) 0 2 1 11 cc c < − =− − + ≤ Nhân theo vế ba bất đẳng thức trên ta được a ab bc c ( ) ( ) ( ) 2 2 21 − − −≤ mẫu thuẫn với (1). Vậy điều phản chứng là sai(đpcm). Bài 4. Giả sử tồn tại một số nhỏ hơn hoặc bằng 0 giả sử là a khi đó do abc > 0 nên a bc < < 0; 0 . Khi đó ab bc ca a b c bc a b c bc b c + + = + + > ⇒ + >− > ⇒ + < ( ) ( ) 0 00 . Suy ra abc ++< 0 mâu thuẫn với giả thiết. Vậy điều phản chứng là sai(đpcm). 21 1  − <  − <. a b Bài 5. Giả sử cả hai phương trình đều vô nghiệm khi đó 4 0 22 2 a b 4 0 Cộng theo vế hai bất đẳng thức trên ta được ( ) 2 2 12 1 2 4 bb a a + >+ . Mặt khác ( ) 2 2 1 2 12 1 2 a a aa b b +≥ ≥ + 2 4 . Suy ra ( ) ( ) 12 12 4 4 bb bb +> + vô lý. Vậy điều phản chứng là sai ta có điều phải chứng minh. Bài 6. Giả sử cả ba phương trình đều vô nghiệm khi đó 2 ∆= − < ∆= − < ' 0 1 b ac 2 . ' 0 2 c ab ∆= − <  2 ' 0 3 a bc Cộng theo vế ba bất đẳng thức trên ta được 222 a b c ab bc ca ++−−−< 0 52 Cty TNHH MTV DVVH Khang Việt ( ) ( ) ( ) 111 222 0 ⇔ −+ −+ −< ab bc ca (vô lý). 222 Vậy điều phản chứng là sai ta có đpcm. Bài 7. Giả sử không có số nào trong ba số f ff (0) , (1) , ( 1) − lớn hơn hoặc bằng 1 11 (0) (1) 2 22   = < −<<   fb b 1 2khi đó 131 (1) 1 (2) 222 . f ab ab   = + + < ⇔ − < + <−   13 1 ( 1) 1 (3) 22 2 f ab ab − = − + + < − <− + <−   Cộng theo vế của (2) và (3) ta được 1 32 12 − < <− ⇒ <− b b mâu thuẫn với (1). Vậy điều phản chứng là sai ta có đpcm. Bài 8. Giả sử tồn tại một số không thuộc đoạn 4 ;0   −   ta có thể giả sử là a khi đó 3 bc a bc a bc a 2 2 2  + =− −   + =− − + =− −     ⇔ ⇔  + =−   + − =− = +  . ( ) ( ) 22 2 2 2 2 bc a b c bc a bc a 2 22 1 Chú ý. ( ) ( ) ( ) 2 22 2 4 4 2 4 1 3 40 0 b c bc a a a a a + ≥ ⇒ − − ≥ + ⇔ + ≤ ⇔− ≤ ≤ mâu thuẫn. 3 Vậy điều phản chứng là sai ta có điều phải chứng minh. Bài 9. Giả sử ngược lại 222 abc + ≥ khi đó ( ) ( ) ( ) 22 222 2 2 20 a b ab bc ca a b c ab bc ca a b c + + + + ≥ + + + + + = ++ ≥ (mẫu thuẫn với giả thiết). Vậy điều phản chứng là sai ta có đpcm. Bài 10. Giả sử tất cả các bất đẳng thức trên là đúng cộng theo vế ba bất đẳng thức 2 ta được ( ) ( ) 9 3 ab bc ca a b c + + ≥ ++ 2 ( ) ( ) a b c ab bc ca 3 0 ⇔ ++ − + + ≤ 111 0 222 ( ) ( ) ( ) ab bc ca abc ⇔ − + − + − ≤⇔== 222 Điều này mâu thuẫn với giả thiết ba số a,b,c đôi một phân biệt. Vậy điều phản chứng là sai và bài toán được chứng minh. Bài 11. Giả sử tất cả các bất đẳng thức là đúng khi đó 53 Khám phá tư duy Kỹ thuật giải bất ĐT Bài toán Max – Min – Đặng Thành Nam abcd +<+ ( )( ) a b c d ab cd (1) . + +<+ ( ) ( ) a b cd c d ab + <+ (2) (3) Nhân theo vế của (1) và (2) ta được 2 2 ( ) ( ) ( )( ) ( ) a b c d c d ab cd a b ab cd + + <+ + ⇔+ < + . 2 2 2 3 3 3 (4) ( ) ⇒ > ++=− + ≥ ⇒ > cd a ab b a b ab ab cd ab Nhân theo vế của (2) và (3) ta có 2 ( ) ( ) ( ) ( ) a b cd c d c d ab ab cd + + <+ + 2 4 ( ) ( ) ( ) . ⇒+ < + ⇒ + > a b cd ab cd ab ab ab cd abcd ⇒+> ⇒> ab cd cd cd ab 4 3 (5) Từ (4) và (5) ta có điều mâu thuẫn. Vậy phản chứng là sai và ta có đpcm. Bài 12. Đặt 1 11 18 18 18 xyz abc = = = + ++ . ; ; Dễ thấy 0 ,, 1 < < xyz và 2 22 xyz abc −−− = = = . 111 ; ; 8 8 8 2 2 x y z Do abc=1 nên ta có 32 22 2 2 2 8 (1 )(1 )(1 ) xyz x y z =− − − . Theo đề bài ta cần chứng minh xyz ++≥1. Giả sử ngược lại xyz ++<1 . Ta có: 2 22 1 ( ) ( )[( ) ( )] 2( ) ( )( 0 − > ++ − = + + + + ≥ + + +> x x y z x z y x y x z y z x yx z . Thiết lập tương tự với hai biểu thức còn lại ta có 2 1 2( ) ( )( ) 0 − > + + +> y x z y zy x . 2 1 2( ) ( )( ) 0 −> + + +> z y x x zz y Nhân lại theo vế ba bất đẳng thức trên ta có: 32 22 2 2 2 2 2 2 8 (1 )(1 )(1 ) 8( ) ( ) ( ) xyz x y z x z y x z y =− − − > + + + ⇒ >+ + + . 8 ( )( )( ) xyz x y z x y z Điều này mâu thuẫn do theo bất đẳng thức AM – GM ta có ( )( )( ) 8 x y z x y z xyz + + +≥ . Do đó ta có điều phải chứng minh. Bài 13. Giả sử 22 2 2 1 min{ ; ; ; } 4 abbccd d a − − − −> . 54 Cty TNHH MTV DVVH Khang Việt Suy ra 222 2 abcd a b c d +++ − + + + > ( )1. Mặt khác: 2 abcd a b c d abcd abcd ( ) ( ) ( )11 4 222 2 +++ + + + − + + + > − + + + +− . abcd +++ 2 ( 1) 1 1 2 = − − >− Suy ra 222 2 abcd a b c d +++ − + + + < ( )1 nên điều giả sử là sai . Suy ra điều cần chứng minh. Bài 14. Giả sử ngược lại abc abc max a b b c c a + +−> − − − { } 3 22 2 ( ) ;( ) ;( ) 3 Suy ra 3 . a b c abc a b c ab bc ca ++− > ++− − − 3 2( ) 3 222 3 ab bc ca a b c abc ⇔ + + >+++ Đổi biến lại 3 33 3 3 3 x y z xyz xy yz zx +++ < + + 3 2( ) 2( ) 2( ) . Nhưng theo bất đẳng thức Schur ta có: 3 33 x y z xyz xy x y yz y z zx z x + + + ≥ ++ ++ + 3 ( )( )() 333 ≥++ 2( ) 2( ) 2( ) xy yz zx Suy ra mâu thuẫn, vậy giả sử sai, suy ra điều cần chứng minh. Bài 15. Đặt 222 ,, 4 abc +++ . x y z xy yz zx xyz bc ca ab = = = ⇒+++ = Ta có xyz x y z xy yz zx ≤ ++≥ + + 1; . Bất đẳng thức trở thành: ( )( )( ) ( )3 2 2 2 21 kx ky kz k + + +≥ + ( ) ( ) 2 2 ⇔ ++ + + + + ≥ + + 4 2 12 6 1 k x y z k xy yz zx xyz k k . 2 Ta có ( ) ( ) 4 2 k x y z k xy yz zx xyz ++ + + + + 2 ( ) ( ) ≥ ++ + ++ + 4 2 k xy yz zx k xy yz zx xyz 2 2 ( )( ) ( )( ) = + ++ + = + − + 4 2 4 24 k k xy yz zx xyz k k xyz xyz 2 2 ( ) = +− +− 16 8 4 2 1 k k xyz k k 22 2 ≥ + − − += + + 16 8 4 2 1 12 6 1 k kk k k k Bất đẳng thức được chứng minh. Đẳng thức xảy ra khi và chỉ khi abc = = 55 Khám phá tư duy Kỹ thuật giải bất ĐT Bài toán Max – Min – Đặng Thành Nam CHỦ ĐỀ 3: KỸ THUẬT QUY NẠP TOÁN HỌC A. NỘI DUNG PHƯƠNG PHÁP Giả sử ta cần chứng minh bất đẳng thức A n() 0 ≥ với A(n) là một biểu thức có chứa số nguyên dương n với 0 n n ≥ . Ta thực hiện chứng minh bằng quy nạp như sau ✔ Chứng minh bất đẳng thức đúng với giá trị đầu tiên của n là 0 n . ✔ Giả sử bất đẳng thức đúng với 0 nkn = > tức A k() 0 ≥ . ✔ Sau đó chứng minh A k( 1) 0 + ≥ dựa vào A k() 0 ≥ . ✔ Kết luận bất đẳng thức đúng. B. BÀI TẬP MẪU Ví dụ 1. Cho n là số nguyên dương n ≥ 5. Chứng minh rằng 2 2n > n . Lời giải + Với 5 2 n = =>= 5;2 32 5 25 . Vậy bất đẳng thức đúng với n = 5 . + Giả sử bất đẳng thức đúng với n k = > 5 tức 2 2k > k . Khi đó ( ) ( ) ( ) ( ) 12 2 22 2 2 2.2 2 1 2 1 1 5 3 1 1 k k k k k k k kk k k + = > = + + − −= + + − + −> + . Vậy bất đẳng thức đúng với n k = +1. Theo nguyên lý quy nạp ta có điều phải chứng minh. Bài tập tương tự Cho n là số nguyên dương lớn hơn 1. Chứng minh ( ) 1 1 n n n n − > + . Bài 2. Cho a,b là các số thực không âm và n là số nguyên dương. Chứng minh rằng n n n a b ab + +   ≥     . 2 2 Lời giải + Với n =1 bất đẳng thức trở thành đẳng thức. k k k a b ab + +   ≥     . + Giả sử bất đẳng thức đúng với n k = >1 tức 2 2 k k k a b ab + + + + +   ≥     . 1 1 1 + Ta cần chứng minh Chú ý 2 2 k k k k a b a b a b a ba b +      + + + ++      = ≤      . 1 . 2 2 2 22 56 Cty TNHH MTV DVVH Khang Việt Vậy ta chứng minh + + + ++ + + ≥ ⇔ + ≥+ 1 11 1 . 2 22 k k kkk k kk a b a ba b a b ab ba kk k kk ( ) ( ) ( )( ) 0 0 ⇔ − + − ≥⇔ − − ≥ aa b b b a a b a b Bất đẳng thức cuối luôn đúng. Vậy theo nguyên lý quy nạp ta có điều phải chứng. Đẳng thức xảy ra khi và chỉ khi a b = . Tổng quát với n số thực không âm và m là một số nguyên dương ta có 1 2 12 ... ... mm m m n n a a a aa a + ++ + ++   ≥     . n n Bài 3. Cho số nguyên dương M > 3. Giả sử 1 2 2014 xx x , ,..., là các số nguyên dương sao cho 1 2 2014 xx x M . ..... . = Tìm giá trị lớn nhất của biểu thức: 33 3 1 2 2014 Sx x x = + ++ ... . Lời giải Ta chứng minh với n là số nguyên dương 1 2 , ,..., n xx x thoả mãn điều kiện 1 2... n xx x M= thì ( ) 33 3 3 1 2 12 ... ... 1 n n x x x xx x n + + + ≤ +− . Và bài toán chính là trường hợp riêng khi n bằng 2014. + Với n bằng 1 bất đẳng thức chính là hằng đẳng thức. + Với n bằng 2 ta có ( )( ) 3 3 3 3 33 1 2 1 2 12 x x x x xx − − ≥ ⇒ + ≤+ 1 10 1 . Vậy bất đẳng thức đúng với n bằng 1,2. + Giả sử bất đẳng thức đúng với n k = > 2 tức ( ) 33 3 3 1 2 12 ... ... 1 k k x x x xx x k + + + ≤ +− (1). Ta cần chứng minh ( ) 33 3 3 1 2 1 12 1 ... ... k k x x x xx x k + ++ ≤ + + + . 3 3 3 Do (1) nên ta chỉ cần chứng minh ( ) ( ) x xx x xx x x + ≤+ ... 1 ... k k kk + + 1 12 12 1 3 3 ( )(( ) ) ⇔ − −≥ x xx x 1 ... 1 0 k k + Bất đẳng thức cuối đúng. 1 12 Theo nguyên lý quy nạp ta có điều phải chứng minh. Vậy giá trị lớn nhất của S bằng 3 M + 2013 đạt tại một số bằng M và 2013 số bằng 1. Bài 4. Cho n là số nguyên dương ≥ = 1, 1, . k x kn Chứng minh rằng 11 1 ... 11 1 1 ... n n n n ++ + + . + ++ ≥ xx x xx x 1 2 1 2 57 Khám phá tư duy Kỹ thuật giải bất ĐT Bài toán Max – Min – Đặng Thành Nam Lời giải + Với n =1 bất đẳng thức trở thành đẳng thức. + Giả sử bất đẳng thức đúng với n k = >1, tức k 11 1 ... 11 1 1 ... k k k ++ + +(1). + ++ ≥ xx x xx x 1 2 1 2 + Ta cần chứng minh 11 1 1 1 ... 11 1 1 1 ... k k k k k + 1 12 1 12 1 ++ + + + . + ++ + ≥ xx x x xx x + + + Do (1) nên ta chỉ cần chứng minh + k k 1 1 1 + + +(2). + ≥ xx x xx x x + + + 1 ... 1 ... k k 1 k k k 12 12 1 1 − + ≥ k k Chú ý 1 1 + + x xx x 1 1 ... + + + − + + + k k k k k k k k k k 1 2 1 12 1 11 1 1 2 1 + ( ) xx x x 1 ... k kk + ≥ 2 + − + + + + k k k k k k k k k k k 2 1 1 12 12 1 + + xx x xx x + 1 1 1 2 1 ( ) xx x x 1 ... 1 ... 1 ... Cộng theo vế hai bất đẳng thức trên ta có (2). Tức bất đẳng thức đúng với n k = +1. Vậy theo nguyên lý quy nạp ta có điều phải chứng minh. Đẳng thức xảy ra khi và chỉ khi 1 2 ... n xx x = = = . Bài 5. (VMO 2011) Chứng minh rằng với mọi số thực dương x và số nguyên dương n ta có ( ) 1 1 2 1 1 n n n + + +   + x x x +   . x n ≤   1 2 Đẳng thức xảy ra khi nào? Lời giải + Với n =1 bất đẳng thức trở thành ( ) 2 3 1 1 +   +   ≥   + x x x 2 1 x ( ) ( ) ( ) 4 4 2 ⇔ + − + ≥⇔ − ≥ x xx x 1 8 10 1 0 . Vậy bất đẳng thức đúng với n =1. + Giả sử bất đẳng thức đúng với n k = tức ( ) 1 1 2 1 1 k k k + + +   + x x x +   (1). k x Ta cần chứng minh ≤   1 2 58 Cty TNHH MTV DVVH Khang Việt + + + +   + 2 3 1 2 1 1 kk ( ) k x x x +   . x + k ≤   1 1 2 Do (1) nên ta chỉ cần chứng minh x + + + 1 2 k k k 1 2 k ( ) ( ) +   +    + xx x x 1 1 1 ≥ 2 k k x x + 1 1 1 + + 2 2 + ++ kk k k k 1 12 ( ) ( ) ( ) ( ) . 4 0 1 1 x xx x x x 1 1 ⇔ + + − + +≥ 2 2 1 k + ( ) ( ) 1 10 x x ⇔− − ≥ Bất đẳng thức luôn đúng. Vậy theo nguyên lý quy nạp ta có điều phải chứng minh. Đẳng thức xảy ra khi và chỉ khi x =1. Để kết thức tôi trình bày lời giải chứng minh bất đẳng thức AM – GM bằng quy nạp. Bài 6. (BĐT AM – GM ) Cho a1, a2, …, an là các số thực không âm. Chứng minh rằng ta luôn có 1 2 12 ... ... n n n a a a n aa a + ++ ≥ Lời giải Cơ sở quy nạp với n = 1, 2 được kiểm tra dễ dàng. Giả sử bất đẳng thức đã được chứng minh cho n số. Xét n+1 số không âm a1, a2, …, an+1. Đặt a1a2…an+1 = An+1. Nếu tất cả các số bằng nhau thì bất đẳng thức đúng. Trong trường hợp ngược lại, phải tồn tại hai số ai, aj sao cho ai < A < aj. Không mất tính tổng quát, có thể giả sử an < A < an+1. Khi đó ta có (an – A)(an+1 – A) < 0, suy ra an + an+1 > anan+1/A + A. Từ đó ta có a1 + a2 + …+ an + an+1 > a1 + … + an-1 + anan+1/A + A(1) Bây giờ áp dụng bất đẳng thức Cauchy cho n số a1 + … + an-1 + anan+1/A ta 1 2 1 12 1 ... ... n n n nn na a a a a a n a a a nA được 1 + + ++ + ≥ = − − . A Kết hợp với (1) ta có điều phải chứng minh. C. BÀI TẬP RÈN LUYỆN Bài 1. Cho a,b,c là độ dài ba cạnh một tam giác vuông có c là cạnh huyền. Chứng minh 222 nnn abc + ≤ với mọi số nguyên dương n. Bài 2. Cho n số thực không âm 1 2 , ,..., n xx x thoả mãn điều kiện ( ) 1 2 2 ... 1 n xx x + ++ ≤ . 59 Khám phá tư duy Kỹ thuật giải bất ĐT Bài toán Max – Min – Đặng Thành Nam Chứng minh rằng ( )( ) 1 2 ( ) 1 1 1 ... 12 n − − −≥ xx x . Bài 3. Chứng minh rằng với n số nguyên dương phân biệt ta có ( ) 33 3 2 1 2 12 ... ... n n a a a aa a + ++ ≥ + ++ . Bài 4. Cho x1, x2, …, xn là các số thực thuộc [0, 1]. Chứng minh rằng x1(1-x2) + x2(1-x3) + … + xn(1-x1) ≤ [n/2] Bài 5. Cho n ≥ 2 và x1, x2, …, xn là n số nguyên phân biệt. Chứng minh rằng (x1-x2)2 + (x2-x3)2 + … + (xn – x1)2 ≥ 4n – 6 Bài 6. Chứng minh rằng với x1 ≥ x2 ≥ … ≥ xn ≥ 0 ta có bất đẳng thức n n i x ∑ ∑≤ . 2 xi = = i i i 1 1 Bài 7. Chứng minh rằng nếu a1, a2, …, an là các số nguyên dương phân biệt thì ta 2 n n   có bất đẳng thức   ∑ ∑ . 75 3 + ≥   ( )2 aa a ii i 1 1 i i = = Bài 8. (Bất đẳng thức Mc-Lauflin) Với mọi số thực a1, a2, …, a2n và b1, b2, …, b2n 2 2 22 2 nn n n       ∑∑ ∑ ∑ . ta có bất đẳng thức 2 22 2 1 2 12 − −≥    a b a b a b ab − − ( ) k k k k k k kk 11 1 1 = = = = kk k k D. HƯỚNG DẪN GIẢI – ĐÁP SỐ Bài 1. Với n =1 bất đẳng thức trở thành đẳng thức. Giả sử bất đẳng thức đúng với n k = >1 tức 222 kkk abc + ≤ (1). Ta cần chứng minh 21 21 21 ( ) ( ) ( ) kkk abc +++ + ≤ . Chú ý do có (1) nên ta chỉ cần chứng minh ( ) 22 2 k k 21 21 ( ) ( ) k k ca b a b + + +≥ + ( )( ) 2 2 2 2 22 22 21 21 ( ) ( ) 0 kk k k k k a b a b a b ab ba ⇔+ + ≥ + ⇔ + ≥ + + . Bất đẳng thức. Vậy theo nguyên lý quy nạp ta có điều phải chứng minh. CHỦ ĐỀ 4: KỸ THUẬT MIỀN GIÁ TRỊ A. GIỚI THIỆU Phương pháp này khá cơ bản và thường được sử dụng trong các bài toán tìm cực trị với các bất đẳng thức có dạng: a fx b ≤ ≤ ( ) với x D∈ . 60 Cty TNHH MTV DVVH Khang Việt Nguyên tắc chung là đưa về tìm điều kiện để phương trình m fx = ( ) có nghiệm trên D. Trong trường hợp có nhiều biến ta cần đưa về phương trình với một biến hoặc đưa về hệ phương trình nếu có. B. NỘI DUNG PHƯƠNG PHÁP 1) Tìm miền giá trị bằng cách xét điều kiện có nghiệm của phương trình bậc hai Một phương trình bậc hai có dạng: 2 Ax Bx C + += 0 với A ≠ 0 . Điều kiện để phương trình có nghiệm là: 2 ∆= − ≥ B AC 4 0 . Vì vậy cần tìm giá trị lớn nhất và giá trị nhỏ nhất của một biểu thức m f xyz = (, ,) . Ta biến đổi tương đương m để đưa về một phương trình bậc của x hoặc y hoặc của z. Khi đó sử dụng điều kiện có nghiệm ta tìm được Max và Min của m. Ví dụ 1. Cho x,y,z là các số thực thoả mãn điều kiện 2 2235  −+=  ++= . xyz xyz Tìm giá trị lớn nhất và nhỏ nhất của biểu thức 2 x y Pz+ − = + . 2 Lời giải Trong biểu thức của P có z khác so với x và y do vậy ta tìm cách rút x y + theo z và đưa về phương trình bậc hai đối với z. Việc tìm Max và Min của P ta chặn bằng điều kiện có nghiệm của phương trình bậc hai đối với z. Ta có ( ) ( ) ( ) 2 2 z Pxy z P xy + = + − ⇒ + +  = + 2 2 22   . Chú ý  − =−  − =−     ⇔ ⇒ + =− + + xy z xy zxy z z 3 3 ( ) ( ) ( )2 2  + =− + + − =−  . 1 1 3 61 22 2 2 2 2 xy z xy xy z 5 5 2 2 Vì vậy ( ) 2 2  + +  =− + +   zP zz 2 2 3 61 2 2 2 ( ) ( ) . ⇔ + + + + − += z P z Pz z 2 4 2 3 6 30 22 2 2 ( ) ( ) ⇔ + + + − + + += P z P P zP P 3 4 4 6 4 8 3 0 (1) Ta có (1) là phương trình bậc hai đối với z điều kiện có nghiệm là 61 Khám phá tư duy Kỹ thuật giải bất ĐT Bài toán Max – Min – Đặng Thành Nam ( ) ( )( ) 2 2 22 ∆= + − − + + + ≥ ' 2 2 3 34 8 3 0 PP P PP 36 23 36 0 0 2 ⇔ + ≤ ⇔− ≤ ≤ PP P 23 + Với xyz = = = 2, 0, 1 ta có P bằng 0. Vậy giá trị lớn nhất của P bằng 0. + Với 20 66 7 , , 31 31 31 xy z = =− = thì P bằng -36/23. Vậy giá trị nhỏ nhất của P bằng 36 − . 23 Bài tập tương tự Cho x,y,z là các số thực thoả mãn điều kiện 2 2235  −+=  ++= . xyz xyz Tìm giá trị lớn nhất và nhỏ nhất của biểu thức 1 x y Pz+ − = + . 2 Đáp số: 3 23 3 23 P − + ≤ ≤ . 3 3 2) Kỹ thuật sử dụng điều kiện có nghiệm của phương trình lượng giác Phương trình dạng: A xB xC sin cos + = có nghiệm khi và chỉ khi 222 ABC + ≥ . x x + + Ví dụ 1. Chứng minh rằng với mọi số thực x ta có 2 cos 2sin 3 2 − + . ≤ ≤ 11 2cos sin 4 x x Lời giải + + Đặt ( ) cos 2sin 3 2cos sin 4 cos 2sin 3 x x − + . = ⇒ − += + + y y xx x x 2cos sin 4 x x ⇔ − − + =− ( ) ( ) 2 1 cos 2 sin 3 4 (1) y xy x y . Điều kiện để (1) có nghiệm là ( ) ( ) ( ) 22 2 2 1 2 34 yy y − + + ≥− 2 11 24 4 0 2 2 ⇔ − +≤⇔ ≤≤ yy y 11 Bài toán được chứng minh. 3) Kỹ thuật điều kiện có nghiệm của phương trình - hệ phương trình Thay vì tìm trực tiếp Max và Min của biểu thức ta đưa về tìm điều kiện có nghiệm của hệ phương trình. Ví dụ 1. Cho x,y là hai số thực thoả mãn điều kiện ( )2 2 2 22 2 2 x y xy x y −+ + −−= 14 0 . 62 Cty TNHH MTV DVVH Khang Việt Chứng minh rằng 35 35 2 2 − + ≤+≤ . 2 2 x y Lời giải Đặt 2 2 mx y = + . Ta cần tìm Max và Min của m. Thay vì đi chứng minh trực tiếp ta tìm điều kiện để hệ phương trình sau có 2 2  = +  −+ + −−=  mx y nghiệm ( ) 2 2 2 22 2 2 14 0 x y xy x y 2 2  = + ⇔  mx y 4 4 2 2 22 3 2 10 x y x y xy  + + − + += 2 2  = +  ⇔  + − + + +=  mx y 2 22 22 2 ( ) ( ) 3 4 10 xy xy x 2  − +  = −    = + 3 1 2 2 mx y x 2 m m 4 ⇔ ⇔    − ++ = + +  =  22 2 m m x mm 3 14 0 1 y 2 4 Điều kiện để hệ phương trình có nghiệm là 2  − + − ≥  − +  ⇔ ≤≤ 3 1 0 m m 4 35 35 . m 2 1 2 2  + +≥  m m 4 0 Vì vậy 35 35 2 2 − + ≤+≤ . 2 2 x y Bài tập tương tự Cho x,y là các số thực thoả mãn điều kiện 22 2 2 x y x y x y xy , 1;4 18 16 ≥ −−= − . Tìm giá trị lớn nhất và giá trị nhỏ nhất của biểu thức x y P + = 2 2 2 . ++ ++++ x xy y xy 5 Đáp số: P P min max =− = − 16 17 4 ; 64 37 6 ( ) ( ). Ví dụ 2. Cho x,y là các số thực thay đổi thoả mãn điều kiện xy x y + − = ++ + 2 21 21 . Tìm giá trị lớn nhất của biểu thức ( )2 P xy = − . 63 Khám phá tư duy Kỹ thuật giải bất ĐT Bài toán Max – Min – Đặng Thành Nam Lời giải 2  −  =   = +   ≥ ⇒ 1 x u 2 1 2 ,, 0 u xu v Đặt ( )  = +  − = . 2 2 1 1 v y v y 2 2 2 2 2 2 2 62 u vuv u v uv + −−=+⇔ + −= + Điều kiện bài toán trở thành ( ) 2 ( ) ( ) 2 ⇔ + − + − −= u v u v uv 2 2 60 . 2 2 2 2 2 2 Khi đó ( )   − − − =− =   u v u v P 1 1 . 22 4   2 2 2 2   +− + − +   = = ( ) ( ) ( ) ( ) 4 u v uv u v uv uv 4 4 2 Vậy ta có hệ phương trình  + − + − −=    +− +    = u v u v uv ( ) ( ) 2 2 6 0 (1) . 2 2 ( ) ( ) u v uv u v 4 P (2) 4 Từ (1) ta có ( ) ( ) ( ) 2 2 1 2 262 uv u v u v u v = + − + −≤ + ( ) ( ) 2 ⇒ + − + − ≤ ⇔− ≤ + ≤ uv uv uv 4 12 0 2 6 . Chú ý uv xy x y xy += + ++ + + ≥ + + ≥ 2 2 2 2 12 1 2 2 6 ( ) ( )( ) ( ) . Vì vậy ta có tuv = +∈  6;6   . 22 2 + − + − +− +     = Khi đó { } ( ) ( ) ( ) ( ) uv uv uv uv 2 26 P 4 2 2 2 2   −+ + ++ + −+ +   = = ( ) ( ) ( ) ( ) uv uv uv tt t 4 12 4 12 4 4 Xét hàm số ( ) 2 2 4 12 f t−+ + = liên tục trên đoạn   6;6   ta có tt t ( ) 4 ( ) 6;6 2 3 33 '( ) 3 6 ; '( ) 0 2 ∈    + = − + + = ←→ = t ft t t t ft t . 64 Cty TNHH MTV DVVH Khang Việt Ta có 3 33 3 69 11 33 ( ) ( 6) 9 6 6; ; (6) 0 2 8   + + ff f . =+ = =     Vì vậy ( ) max max3 69 11 33 3 33 2 8 Pf f  + + = = =   .   Dấu bằng xảy ra khi và chỉ khi   + +   += + = 3 33 7 33 uv xy 2 2   ⇔ 3 33 3 33 2 21 21   + + + − = ++ +=   xy x y 2 2  +−+ +++  = = 14 2 33 414 66 33 14 2 33 414 66 33 , 8 8 x y ⇔ +++ +−+  = =  14 2 33 414 66 33 14 2 33 414 66 33 , 8 8 x y Vậy giá trị lớn nhất của P bằng 3 69 11 33 ( ) + . 8 Ví dụ 3. (TSĐH Khối A 2006) Cho x,y là 2 số thực và x y ≠ ≠ 0, 0 thỏa mãn điều kiện 2 2 xy x y x y xy ( ) +=+ − . 1 1 Ax y Tìm giá trị lớn nhất của biểu thức 3 3 = + . Lời giải Nhận thấy x, y đối xứng nên đặt: .xyu  + =  = . xy v u 2 Giả thiết bài toán trở thành: 2 . 33 = − ⇔=+(do u ≠ −3 ) uv u v vu 33 2 2 2 1 1 ( 3) 3 x y uu v u u +− +   += = ==     Ta có ( ) 33 3 3 2 xy v v xy u 2 2 2 44 1 1 u u u Vì −  ≥ ≥ ⇒ ≥ ⇔ ≤⇔ ≥⇔  ++ +  ≤ − . u vuuu u u 4 10 33 3 3 Chú ý để tìm Max của A ta chỉ cần xét với 3 0 uu+> nên ta chỉ cần chứng minh Max của A bằng 16 vậy : 3 4 uu+≤ với u ≥1 hoặc u ≤ −3. 65 Khám phá tư duy Kỹ thuật giải bất ĐT Bài toán Max – Min – Đặng Thành Nam − < +⇒f’(u) = 23 0 Xét hàm số f(u) = u 3 u u Trên mỗi khoảng ( −∞ −; 3 ) và [1; ) +∞ do đó f(u) ≤ f(1); ∀ ≥ u 1. Còn 0 < f(-3) < f(u) <1, ∀ u > -3. Vậy giá trị lớn nhất của A bằng 16. C. BÀI TẬP RÈN LUYỆN Bài 1. Chứng minh rằng với mọi số thực x ta có 1 2sin cos 1 2 x x + + − + . −≤ ≤ 2 sin 2cos 3 x x Bài 2. Chứng minh rằng với mọi số thực x và y ta có 2 2 2 cos3 sin 3 1 3 2 23 1 xy x y y   + + ++ +   ≤   + . cos3 2 9 x Bài 3. Chứng minh rằng với mọi số thực x,y thoả mãn điều kiện ( )2 2 2 2 2 x y x y x y+ + 2 2 5 3. − += − . 42 2 Chứng minh rằng 55 55 2 2 − + ≤+≤ . 2 2 x y Bài 4. Cho x,y là các số thực thoả mãn điều kiện 2 2 x xy y ++ ≤ 3. Chứng minh rằng 2 2 − −≤ − − ≤ − 43 3 3 43 3 x xy y . Bài 5. Cho x,y,z là các số thực thoả mãn điều kiện 2 2235  ++=  ++= . xyz xyz Tìm giá trị lớn nhất, giá trị nhỏ nhất của biểu thức 3 x y Pz− + = + . 2 Bài 6. Cho x,y là hai số thực thay đổi thoả mãn điều kiện 2 3 34 x y ++ += . Tìm giá trị lớn nhất và giá trị nhỏ nhất của biểu thức Px y = ++ + 2 9 . D. HƯỚNG DẪN GIẢI – ĐÁP SỐ x x + + Bài 1. Đặt ( ) ( ) 2sin cos 1 2 sin 2 1 cos 3 1 − + . y y x y xy = ⇒− + + = − sin 2cos 3 x x Điều kiện để phương trình có nghiệm là:( ) ( ) ( ) 222 2 21 31 − + +≥ − yy y 2 1 4 6 40 2 ⇔ − − ≤ ⇔− ≤ ≤ yy y . 2 Bài toán được chứng minh. 66 Cty TNHH MTV DVVH Khang Việt xy x m m xy x m + + Bài 2. Đặt ( ) cos3 sin 3 1 1 cos3 sin 3 1 2 + . = ⇒ − − =− cos3 2 x Điều kiện để phương trình có nghiệm là ( ) ( ) 2 2 2 my m − + ≥− 1 12 2 2 2 2 13 1 13 1 32 03 3 −+ ++ y y ⇔ − − ≤⇔ ≤ ≤ . m my m 2   + + + + ++ + 2 22 2 1 3 1 1 3 1 3 2 23 1 2 y yyy Suy ra ≤ ⇒≤ =      . m m 3 39 Bài toán được chứng minh. 2  − +  = −  − + = . m m x Bài 3. Tìm được: 2 2 5 5 4 y 2 m m 4 5 Điều kiện để hệ phương trình có nghiệm là: 2  − + − ≥  − +  ⇔ − +≤⇔ ≤ ≤ 5 5 0 m m 4 55 55 5 50 2 . mm m 2 5 2 2  − +≥  m m 4 0 Bài toán được chứng minh. Bài 4. Nếu y = 0 bất đẳng thức luôn đúng. 2 22 x xy y t t mx xy y t t − − −− = = + + ++ . Xét với y ≠ 0 đặt 3 3 2 22 1 Suy ra ( ) ( ) ( ) 22 2 mt t t t m t m t m ++ = −−⇔ − + + + + = 1 3 1 1 30 . Điều kiện để phương trình có nghiệm là ( ) ( )( ) 2 ∆= + − − + ≥ m mm 1 4 1 30 −− −+ 2 3 43 3 43 3 6 13 03 3 ⇔ − − ≤⇔ ≤ ≤ . mm m Chú ý do 22 2 2 0 3 43 3 3 43 3 < + + ≤ ⇒− − ≤ − − ≤ − x xy y x xy y . Bài toán được chứng minh. Bài 5. Chú ý z ≠ −2 vì ngược lại ta có 2 251  + =  + = vô nghiệm. x y x y Khi đó ( ) ( ) z Pxy z P xy + =−+⇔ + −=− 2 3 23 . Từ hệ điều kiện ta có 67 Khám phá tư duy Kỹ thuật giải bất ĐT Bài toán Max – Min – Đặng Thành Nam  + =−  + =−     ⇔ ⇒ − =− + + xy z xy zxy z z 3 3 ( ) ( ) ( )2 2  + =− + + − =−  . 1 1 3 61 22 2 2 2 2 xy z xy xy z 5 5 2 2 Khi đó ( ) 2 2  + −  =− + +   zP zz 2 3 3 61 2 2 2 ( ) ( ) ⇔ + − + + − += z P z Pz z 2 6 2 3 6 80 22 2 2 ( ) ( ) ⇔ + + − − + − += P z P P zP P 3 4 6 6 4 12 8 0 Điều kiện phương trình có nghiệm là ( ) ( )( ) 2 2 22 4 3 3 3 4 12 8 0 PP P P P − − − + − +≥ 27 8 6 27 8 6 23 54 15 023 23 − + 2 ⇔ − + ≤⇔ ≤ ≤ PP P P P − + = = . 27 8 6 27 8 6 Vì vậy min max ; 23 23 2 3 10 22; 2 2 P P =+ = . Bài 6. Thực hiện tương tự bài tập mẫu: max min CHỦ ĐỀ 5: KỸ THUẬT SỬ DỤNG NGUYÊN LÝ DIRICLE BÀI TẬP MẪU Bài 1. Cho x,y,z là các số thực dương thoả mãn điều kiện x y z xyz + + +=1 4 . Chứng minh rằng xy yz zx x y z + + ≥++ . Lời giải Theo nguyên lý Dirichlet tồn tại 2 trong 3 số ( 1),( 1),( 1) xyz −−− cùng dấu, không mất tính tổng quát giả sử ( )( ) x y xy x y − − ≥⇔ ≥+− 1 10 1. ⇒ + + ≥ + −+ + xy yz zx x y yz zx 1 . Vậy ta cần chứng minh x y yz zx x y z z x y + −+ + ≥ + + ⇔ + − ≥ 1 11 ( ) . Theo giả thiết ta có 1 zxy+ + = − . x y 4 1 Vậy ta chứng minh + ++ − ≥⇔ + −≥ −⇔ − ≥ − . x y x y x y xy x y ( ) ( ) ( ) 1 2 2 . 1 1 14 1 0 xy 4 1 Bất đẳng thức được chứng minh. Đẳng thức xảy ra khi và chỉ khi xyz = = =1. 68 Cty TNHH MTV DVVH Khang Việt Bài tập tương tự Cho a,b,c là các số thực dương thoả mãn điều kiện a b c abc +++ = 4 . Chứng minh rằng a b c ab bc ca ++≥ + + . Bài 2. Cho a,b,c là các số thực không âm thỏa mãn điều kiện abc ++= 3 . Chứng minh rằng ( )( )( ) 222 111 1 −+ −+ −+ ≥ aa bb cc . Lời giải Trong ba số 1 ,1 − − a b và 1− c luôn tồn tại hai số cùng dấu không mất tính tổng quát giả sử hai số đó là 1− b và 1− c suy ra ( )( ) 11 0 − −≥ b c . 2 2 2 2 22 Khi đó ( )( ) ( )( ) 11 1 bb cc b bc c b c bc −+ −+ = − − + + −−+ ( )( ) 2 2 bc b c b c b c 11 1 = − − + + −−+ 1 1 1 2 2 2 ( ) ( ) b c bc bc bc ≥ + − −+≥ + − + + 2 2 1 45 3 31 a a 2 ( ) ( ) a a = − − − += − + 2 2 Vì vậy bất đẳng thức được chứng minh nếu ta chứng minh được ( ) ( ) ( ) 2 2 2 4 5 2 1 . 1 1 330 a a − + − + ≥⇔ − − + ≥ (luôn đúng). aa a a a 2 Đẳng thức xảy ra khi và chỉ khi abc = = =1. Nhận xét. Ngoài ra bài toán có thể giải bằng kỹ thuật dồn biến xem chương 4. Bài 3. Cho x,y,z là các số thực không âm thoả mãn điều kiện 2 22 x y z xyz +++ = 2 1. Tìm giá trị lớn nhất của biểu thức P xy yz zx xyz =++− 2 . Lời giải Trong 3 số (2 1),(2 1),(2 1) xyz −−− luôn tồn tại hai số cùng dấu, không mất tính tổng quát giả sử (2 12 1 0 2 4 1 2 )( ) ( ) ( ) 2z x y x y xy z x y xyz − − ≥ ⇒ + − ≤⇒ + − ≤ . Chú ý ( ) 2 22 1 1 2 22 2 12z z xyz x y xy xyz xy z xy − − = + + ≥ + = +⇒ ≤ . Vì vậy 1 1 z z P − ≤ += . 2 22 Với 12 xyz = = = thì P bằng 1 .2 Vậy giá trị lớn nhất của P bằng 1 .2 69 Khám phá tư duy Kỹ thuật giải bất ĐT Bài toán Max – Min – Đặng Thành Nam Bài tập tương tự Cho a,b,c là các số thực dương thoả mãn điều kiện 222 a b c abc +++ = 4 . Chứng minh rằng ab bc ca abc ++− ≤ 2 . Bài 4. Cho x,y,z là các số thực dương có tích bằng 1. 111 2 1 Chứng minh rằng ( ) ( ) ( ) ( )( )( ) 2 22 +++ +++ . +++ ≥ xyz 111 xyz 111 Lời giải Trong ba số ( 1),( 1),( 1) xyz −−− luôn có hai số cùng dấu, không mất tính tổng quát giả sử ( )( ) ( ) ( )( ) x y xy x y xy x y − − ≥ ⇒ +≥ + ⇒ + ≥ + + 1 10 1 2 1 1 1 . 1 11 + + + . Sử dụng bất đẳng thức ( ) ( ) 2 2 + ≥ x y 1 1 1 xy Chứng minh xem chương 3. 1 1 1 1 1 z z Suy ra( ) ( )( ) ( ) ( ) 2 22 ≥ . VTyz z z zz + + =+ + = 1 11 1 1 + ++ + + ++ xy x 1 1 Bất đẳng thức được chứng minh. Đẳng thức xảy ra khi và chỉ khi xyz = = =1. Bài 5. Cho a,b,c là các số thực không âm. Chứng minh ( ) ( ) ( ) 1 222 2 111 ++ − + − + − ≥++     abc a b c a b c . 2 Lời giải Luôn tồn tại hai trong ba số ( 1),( 1),( 1) abc −−− cùng dấu, không mất tính tổng quát giả sử ( )( ) a b ab a b − − ≥⇒ ≥+− 1 10 1. Khi đó ta chỉ cần chứng minh 1 12 1 1 1 222 +− ++ − + − + − ≥++     ( ) ( ) ( ) ( ) ca b a b c a b c 2 222 ( ) ( ) ( ) ( )( ) ⇔ − + − + − ≥ +− − a b c ab c 1 1 1 2 21 Sử dụng bất đẳng thức C –S và bất đẳng thức AM – GM ta có 2 2 ( ) ( ) ( ) ( ) ( ) ( ) ( ) ab ab +− +− 222 2 2 2 2 abc c c − +− +− ≥ +− ≥ − 1 1 1 12 .1 2 2 ( )( ) ( )( ) = +− − ≥ +− − 2 212 21 ab c ab c Bài 6. Cho a,b,c là các số thực không âm chứng minh ( ) 222 a b c abc ab bc ca + + + +≥ + + 2 12 . 70 Cty TNHH MTV DVVH Khang Việt Lời giải Bài toán đã được trình bày trong chủ đề Kỹ thuật sử dụng tam thức bậc hai. Dưới đây là lời giải tiếp cận theo nguyên lý Dircihlet. Luôn tồn tại 2 trong 3 số ( ) ( ) ( ) abc −−− 1, 1, 1 cùng dấu, không mất tính tổng quát ta giả sử ( )( ) ( )( ) ( ) a b c a b abc ac bc c − − ≥⇒ − − ≥⇔ ≥ + − 1 10 2 1 10 2 2 . Vậy ta chỉ cần chứng minh 222 ( ) ( ) a b c ac bc c ab bc ca + + ++ + − ≥ + + 12 2 2 2 ( ) ( ) ⇔ − +− ≥ ab c 1 0 Bất đẳng thức được chứng minh. Đẳng thức xảy ra khi và chỉ khi abc = = =1. Bài toán tương tự Cho a,b,c là các số thực không âm chứng minh ( ) 2 2 2 222 a b c a b c ab bc ca + + + +≥ + + 2 2 . Bài 7. Cho a,b,c là các số thực không âm thoả mãn điều kiện 222 a b c abc +++ = 4 . Chứng minh rằng abc ab bc ca abc ≤++≤ + 2 . Lời giải Bất đẳng thức vế trái đơn giản bởi trong 3 số có ít nhất một số không vượt quá 1 giả sử là c khi đó ab bc ca abc ab c c a b + + − = −+ +≥ ( ) ( ) 1 0 . Đẳng thức xảy ra chẳng hạn tại a bc = = = 2, 0 . Ta chứng minh bất đẳng thức vế phải: Trong ba số ( 1),( 1),( 1) abc −−− luôn có hai số cùng dấu, không mất tính tổng quát giả sử ( )( ) ( ) a b ab a b abc c a b − − ≥ ⇒ ≥ + −⇒ ≥ + − 1 10 1 1 . Vậy ta chứng minh c a b ab bc ca ab c ( ) +− +≥ + + ⇔ ≤− 12 2 . Chú ý 22 2 2 4( ) 2 2 = + + + ≥ + + ⇒ ≤− a b c abc ab c abc ab c . Bất đẳng thức được chứng minh. Bài 8. Cho a,b,c là các số thực dương có tích bằng 1. 111 1 1 Chứng minh rằng ( ) ( ) ( ) 222 +++ +++ . +++ ≥ 111 abc abc 1 Lời giải Theo nguyên lý Dirichlet thì 2 trong 3 số ( ) ( ) ( ) abc −−− 1, 1, 1 cùng dấu, không mất tính tổng quát ta giả sử ( )( ) 1 1 101 c a b ab a b + − − ≥ ⇒+ = ≥ + . c 1 11 c Chú ý. ( ) ( ) 2 2 + + + + . + ≥= 1 1 1 1 a b ab c 71 Khám phá tư duy Kỹ thuật giải bất ĐT Bài toán Max – Min – Đặng Thành Nam 1 1 1 c Pc c c c Gọi P là biểu thức vế trái ta có( )2 + + + + +. ≥+ + = 1 1 1 1 c Bất đẳng thức được chứng minh. Đẳng thức xảy ra khi và chỉ khi abc = = =1. A. BÀI TẬP RÈN LUYỆN Bài 1. Cho a,b,c là các số thực thoả mãn điều kiện 1 1 11 +++ . ++= 222 abc 84 89 8 3 Tìm giá trị lớn nhất và giá trị nhỏ nhất của biểu thức Pa b c =+ + 2 3 . B. HƯỚNG DẪN GIẢI – ĐÁP SỐ 1 1 11 Bài 1. Để đơn giản đặt x ay bz c = = = , 2, 3 ta có 2 22 +++ . ++= xyz 888 3 Ta cần Max và Min của Pxyz =++ . Theo giả thiết kết hợp sử dụng bất đẳng thức AM – GM ta có 11111 =− +− 2 22 8 88 6 6 xyz + ++ 2 2 22 2 21 2 2 . 6 86 8 3 8 8 y z yz + + ++ =+≥ 2 2 2 2 ( ) ( ) y z y z + + + + Tương tự ta có 22 22 xz yx ++ ++ 1 1 2 21 1 2 2 ≥ ≥ .; . 2 22 2 22 8 88 8 88 3 3 y xz z yx + ++ + ++ Nhân theo vế ba bất đẳng thức trên ta được: ( )( )( ) 2 22 xyz + + +≤ 2 2 2 27 (1). Mặt khác ( )( )( ) ( ) 2 22 2 x y z xyz + + + ≥ ++ 2 2 23 (2). Thật vậy bất đẳng thức đã cho tương đương với: ( ) ( ) 2 22 2 2 22 22 x y z x y y z z x xy yz zx + + + +≥ + + 2 86 . Trong ba số 2 22 ( 1),( 1),( 1) xyz −−− luôn có hai số cùng dấu, không mất tính tổng quát giả sử ( )( ) ( ) 2 2 2 2 2 2 2 22 2 2 2 x y xy x y xyz z x y − − ≥ ⇒ ≥ + −⇒ ≥ + − 1 10 1 1 . 72 Cty TNHH MTV DVVH Khang Việt Do đó ta chỉ cần chứng minh 2 2 2 2 2 2 2 2 22 22 ( ) ( ) ( ) z x y x y z x y y z z x xy yz zx + − + + + + + + +≥ + + 1 2 86 22 22 ( ) ( ) ( ) ( ) ⇔− + − + − + − ≥ x y yz xy xz 3 12 13 10 Bất đẳng thức cuối đúng suy ra điều phải chứng minh. Kết hợp (1) và (2) ta có ( )2 xyz xyz + + ≤ ⇔− ≤ + + ≤ 93 3 . Vậy giá trị nhỏ nhất của P bằng -3 đạt tại 1 1 1, , 2 3 ab c =− =− =− . Giá trị lớn nhất của P bằng 3 đạt tại 1 1 1, , 2 3 ab c = = = . CHỦ ĐỀ 6: KỸ THUẬT SỬ DỤNG TAM THỨC BẬC HAI Các tính chất về nghiệm và dấu của tam thức bậc hai có một ứng dụng hết sức sâu rộng trong giải toán. Dưới đây tôi trình bày một ứng dụng của tam thức bậc hại trong chứng minh bất đẳng thức. Bất đẳng thức tiếp cận bằng phương pháp này có thể nói là rất tự nhiên. A. NỘI DUNG PHƯƠNG PHÁP Xét tam thức bậc hai 2 f x ax bx c ( ) = ++ . Đính lý 1. Nếu a > 0 thì f(x) đạt giá trị nhỏ nhất tại 0 2b xa = − và giá trị nhỏ nhất 2 b ac f xa− = − . 4 ( ) 4 0 Định lý 2. Nếu a < 0 thì f(x) đạt giá trị lớn nhất tại 0 2b xa = − và giá trị lớn nhất 2 b ac f xa− = − . 4 ( ) 4 0 Định lý 3. Nếu 2 ∆= − ≥ b ac 4 0 thì f(x) luôn có nghiệm.  > ∆= − ≤thì f x() 0 ≥ với mọi x. a 0 Định lý 4. Nếu 2 b ac 4 0  < ∆= − ≤thì f x() 0 ≤ với mọi x. a 0 Định lý 5. Nếu 2 b ac 4 0 73 Khám phá tư duy Kỹ thuật giải bất ĐT Bài toán Max – Min – Đặng Thành Nam B. BÀI TẬP MẪU Bài 1. Chứng minh rằng với x,y,z là các số thực có tổng bằng 1 ta có ( ) ( ) 2 3 4 5 44 x y z xy yz zx + + ≥ ++ . Lời giải Thay z xy =− − 1 bất đẳng thức trở thành: 2 ( ) ( )( ) 3 4 5 5 5 44 44 1 x y x y xy x y x y + +− − ≥ + + −− 2 2 ( ) ⇔ + −+ − + ≥ 48 16 3 4 45 54 25 0 x xy y y Vế trái là tam thức bậc hai của x với hệ số của 2 x dương và có ( ) ( ) ( ) 2 2 2 ' 64 3 4 48 45 54 25 176 3 1 0 x ∆ = − − − + =− − ≤ y yy y . Vậy bất đẳng thức được chứng minh. Đẳng thức xảy ra khi và chỉ khi 111 , , 236 xyz = = = . Cách 2: Bất đẳng thức đã cho tương đương với: 222 9 16 25 20 4 14 x y z xy yz zx + + ≥ ++ . Sử dụng bất đẳng thức AM – GM ta có 5 5 4 9 2. 4 .9 20 2 2 22 ( ) x y x y xy +≥ ≥ 3 3 7 7 4 36 2. 4 .36 14 2 2 22 ( ) x z x z xz +≥ ≥ 12 12 2 2 22 y z y z yz +≥ ≥ 4 2 .4 4 Cộng theo vế ba bất đẳng thức trên ta có đpcm. Như vậy không cần giả thiết bài toán ba số có tổng bằng 1(Xem thêm chủ đề kỹ thuật tham số hoá – Chương 2). Bài 2. Chứng minh rằng với mọi số thực a và b ta luôn có ( )( ) ( ) 2 2 22 31 1 21 −+ −+ ≥ − + a a b b ab a b . Lời giải Viết lại bất đẳng thức dưới dạng: ( ) ( ) 2 22 2 a a b a a ba a − + − − + + − +≥ 3 3 3 5 3 3 3 10 . Vế trái là tam thức bậc hai của b có 2 aa a − + > ∀∈ 3 3 0,  và ( ) ( )( ) ( ) 2 2 2 22 2 3 5 3 4 3 3 3 3 1 3 1 0, b ∆ = − + − − + − + =− − + ≤ ∀ ∈ aa aa aa aa a  Do đó vế trái luôn không âm. Bài toán được chứng minh. 74 Cty TNHH MTV DVVH Khang Việt Đẳng thức xảy ra khi và chỉ khi Bài tập tương tự 2  − +=  ±  − + ⇔== =  − + . a a 3 10 3 5 3 532 2 a a a b ba a 2 ( ) 2 33 Chứng minh rằng với mọi số thực a,b,c,d ta có 2 2 2 2 22 22 3( )( ) 2( ) a ab b c cd d c a abcd b d −+ −+ ≥ − + . Ta cùng xét một số bài toán cùng dạng sau đây Bài 2.1. Cho a,b,c là các số thực không âm. Chứng minh rằng ( )( )( ) 2 2 2 222 31 1 1 1 − + − + −+ ≥+ + a a b b c c abc a b c . Lời giải Ta có ( )( ) ( ) ( ) ( ) 2 2 22 2 22 21 1 1 1 1 − + − + =+ + − + − − a a b b ab a b a b . Suy ra ( )( ) 2 2 22 21 1 1 − + − + ≥+ a a b b ab . Do ta chỉ cần chứng minh bất đẳng thức sau ( )( ) ( ) 22 2 222 31 1 21 + −+ ≥ + + a b c c abc a b c . ( ) ( ) 22 2 22 22 ⇔ + − + + ++ ≥ 3 32 3 13 0 a b c ab a b c a b . Coi vế trái là tam thức bậc hai của c có hệ số của 2 c dương và ( ) ( )( ) ( ) 2 22 22 22 4 ∆= + + − + + =− − ≤ 3 2 3 43 1 3 31 0 ab a b a b a b ab . Do đó vế trái luôn không âm. Bài toán được chứng minh đẳng thức xảy ra khi và chỉ khi abc = = =1. Bài 2.2. Cho x và y là hai số thực không cùng dương ta luôn có ( )( ) 4 2 2 22 11 1 3 x x y y x y xy −+ −+ ≥ − + . Lời giải Theo giả thiết trong hai số luôn có một số không dương không mất tính tổng quát ta giả sử y ≤ 0 khi đó viết lại bất đẳng thức dưới dạng: ( ) ( ) ( ) 2 2 2 2 y x yyx y − − − + +− ≥ 2 4 7 4 12 0 . Coi vế trái là tam thức bậc hai của x ta có ( ) ( ) ( ) ( ) 2 2 2 2 2 ∆= − + − − − = − + ≤ ∀ ≤ 4 7 4 4 2 1 2 24 51 24 0, 0 y y y y yy y y . 75 Khám phá tư duy Kỹ thuật giải bất ĐT Bài toán Max – Min – Đặng Thành Nam Bài toán được chứng minh đẳng thức xảy ra khi và chỉ khi 1 , 0 2 x y = = hoặc 1 0, 2 x y = = . Mở rộng bài toán cho 3 biến ta có kết quả Cho x,y,z là các số thực không cùng dương ta luôn có ( )( )( ) 16 2 2 2 2 22 1 1 11 9 x x y y z z xyz x y z − + − + − + ≥− + . Bài 3. Cho a,b,c là các số thực không âm thỏa mãn điều kiện 222 abc ++= 2 . Chứng minh rằng: 1 2 + ≥++ abc ab bc ac . Lời giải Giả sử { } 2 c abc c = ⇒≥ . max , , 3 Khi đó đặt S a b P ab =+ = , theo giả thiết bài toán ta có 2 2 S c S Pc P + − 2 2 2 2 22 − + =⇒ = . 22 22 2 2 1 2. . Sc Sc c cS Ta cần chứng minh +− +− + ≥+ 2 2 ( ) 2 32 2 1 2 2 4 40 c S cS c c c ⇔ − − + − − +≥ Vế trái là tam thức bậc hai với hệ số dương do vậy ta chỉ cần chứng minh 2 32 ( )( ) ∆= − − − − + ≤ S c c cc c ' 2 12 4 4 0 2 2 ( ) ( ) ⇔ − +− ≥ c cc 4 1 10 Bất đẳng thức cuối luôn đúng do 23 c ≥ . Bất đẳng thức được chứng minh. Đẳng thức xảy ra khi và chỉ khi ab c = = = 1, 0 hoặc các hoán vị. Bài tập tương tự Cho a, b, c là các số thực dương thoả mãn điều kiện ( 1)( 1)( 1) 1 4 a b c abc + + + =+ . Chứng minh rằng: a b c abc + + ≤+1 . Bài 4. Cho a,b,c là các số thực không âm thay đổi thỏa mãn điều kiện abc ++=1. Tìm giá trị lớn nhất của biểu thức ( ) ( ) ( ) 444 P ab c bc a ca b = −+ −+ − . 76 Cty TNHH MTV DVVH Khang Việt Lời giải Không mất tính tổng quát giả sử abc ≥ ≥ , khi đó 4444 4 4 ( ) ( ) ( ) ( ) = −+ −+ −≤ ++ + P a b c b c a c a b a b c ba ca 4 3 4 3 = + + += + + +     ( ) ( ) ( ) ( ) ab c a b c ab c b c a 3 = + ++ − + ++     . ( ) ( ) ( )( ) ab c a b c ab c a b c 3 2 ( ) ( ) ( ) ( ) ( ) ab c ab c ab c ab c 13 3 = + − + =−  +  + +   2 1 11 36 12 12   =− + − + ≤     ( ) ab c Vậy giá trị lớn nhất của P bằng 112 đạt tại 36 36 , ,0 6 6 abc + − = = = hoặc các hoán vị. Chú ý. Ta có bất đẳng thức mạnh hơn như sau ( ) ( ) ( ) ( ) 444 4 112 ab c bc a ca b a b c + + + + + ≤ ++ Bài 5. Cho các số thực thay đổi x,y,z thõa mãn điều kiện 2 22 16 3 25 x y z xy +++ = . Tìm giá trị nhỏ nhất của biểu thức 3 5 22 2 ( ) 10( ) 5 6 P x y z xy xy yz zx = + + +− ++ . Lời giải Nhận xét. Trước hết ta chưa quan tâm đến điều kiện của x,y,z mà để ý đến tính chất đối xứng của x và y trong điều kiện cũng như biểu thức của P nên ta sử dụng đánh giá: 2 x y x y+ 2 2 ( ) + ≥ . 2 Sử dụng bất đẳng thức AM-GM ta được: 3 5 ( ) 10( ) 10 6 2 2 P x y z xy xy yz zx ≥ + + +− ++ 3 5 2 . 10 2 2 ( ) ( ) ≥ + +− ++ x y z xy xy yz zx 10 6 = + +− ++ ≥++− ++ ( ) 10( ) 10( ) x y z xy xy yz zx xy yz zx xy yz zx Một đánh giá dễ nhận ra là đưa về tam thức bậc 2: 77 Khám phá tư duy Kỹ thuật giải bất ĐT Bài toán Max – Min – Đặng Thành Nam 2   5 55 102 22 xy yz zx xy yz zx xy yz zx ( ) . + + − + + = + + − − ≥−     Đẳng thức xảy ra khi và chỉ khi 5  ++=     =− =− =−  = ⇔     +++ =  = = =    + ≥ . xy yz zx 2 55 2 , ,3 xyz x y 34 34 17 2 22 16 55 2 3 , ,3 25 34 34 17 x y z xy xyz ( ) zx y 0 − đạt tại 55 2 , ,3 Vậy giá trị nhỏ nhất của P bằng 52 55 2 , ,3 34 34 17 xyz = = = . 34 34 17 − =− =− y z hoặc Bài 6. Cho x,y,z là các số thực thỏa mãn điều kiện 2 22 xyz ++= 3 . Tìm giá trị lớn nhất và giá trị nhỏ nhất của biểu thức P xy yz yz =++ 2 . Lời giải Ta tìm giá trị nhỏ nhất của P Xuất phát từ bất đẳng thức cơ bản: 2 22 xyz x y z xy yz zx+ + ( ) 2 3 02 2 + + ≥ ⇔ + + ≥− =− . 22 2 22 33 3 3 xz xyz P xy yz zx zx zx + ++ Suy ra ( ) = + + + ≥− + ≥− − ≥− − =− . 2 22 2 2 Đẳng thức xảy ra khi và chỉ khi    ++=  =− = =  ++= ⇔    + ++  = = = −  =− = =−  . 3 3 0 , 0, xyz x yz 2 22 xyz 2 2 33 3 22 2 22 xz xyz x yz 3 , 0, xz 2 22 2 2 Vậy giá trị nhỏ nhất của P bằng −3 đạt tại 3 3 , 0, 2 2 x yz =− = = hoặc 3 3 , 0, 2 2 x yz = = = − . Cách 2: Viết lại biểu thức P dưới dạng ( ) ( ) ( ) ( ) 2 2 2 2 P x z yx z y x z yx z y P += + + + + ⇔ + + + + − −= 3 3 0 (1) . 78 Cty TNHH MTV DVVH Khang Việt Coi (1) là phương trình bậc 2 với ẩn là ( ) x z + ta có điều kiện có nghiệm của phương trình là ( ) 2 y ∆ = − − − ≥ ⇔ ≥− + ≥− + y yP P . 3 4 30 3 3 2 2 4 x z ( )  =    =− = =  + =− = ⇔      = = = −  ++= . 0 3 3 , 0, yx yz Đẳng thức xảy ra khi và chỉ khi y x z 2 2 0 2 3 3 , 0, 3 2 2 Ta có kết quả tương tự cách trên. Tìm giá trị lớn nhất của P 2 22 xyz x yz Ta tìm số thực k > 0 nhỏ nhất sao cho ( ) 2 22 P xy yz zx k x y z =++ ≤ + + 2 đúng với mọi x,y,z và 2 22 xyz + + . Khi đó giá trị lớn nhất của P bằng 3k. Viết lại bất đẳng thức trên dưới dạng: ( ) 2 22 ky y x z kx kz xz − ++ + − ≥ 2 0 . Đây là một tam thức bậc hai với hệ số của 2 y dương nên ta chỉ cần tìm k sao cho 0 ∆ ≤y . Tức ( ) ( ) ( )( ) ( ) 2 2 2 222 x z k kx kz xz k x z xz k x z + − + − ≤⇔ − + + + ≤∀ 4 2 0 1 4 2 1 4 0, , . Đây là một bất đẳng thức đối xứng với x và z nên ta chọn x z = =1.  −  ≤ k 1 3 2 14 14 0 2 2 101 3 Suy ra 2 2 − ++ ≤ ⇔ − −≥ ⇔  +  ≥ . k k kk k 2 Vậy 1 3 31 3 ( ) k P+ + = ⇒≤ . 2 2 Đẳng thức xảy ra khi và chỉ khi 33 33 , 4 2 xz y + − = = = . Nhận xét. Ngoài lời giải bằng tam thức bậc hai như trên ta có thể sử dụng kỹ thuật tham số hóa hoặc sử dụng C –S. Bài 7. Cho a,b,c là các số thực thuộc đoạn 1;3 .       Chứng minh 3 abc 7 +++ . ++≥ ab bc ca 5 79